[Ответить в тред] Ответить в тред

Check this out!


<<
Назад | Вниз | Обновить тред | Автообновление
540 | 43 | 205

Тупых вопросов тред Аноним 30/05/18 Срд 23:03:09  439790  
15232996422620.png (114Кб, 636x440)
Предыдущий:
https://2ch.hk/sci/res/438003.html
Аноним 30/05/18 Срд 23:04:29  439791
govno.mp4 (250Кб, 444x250, 00:00:07)
И сразу: поясните релейтед, только в виде формулы, пожалуйста.
Аноним 30/05/18 Срд 23:16:23  439794
>>439791
В виде формулы чего конкретно?
Аноним 31/05/18 Чтв 00:21:30  439797
Гугли брахистохрона, циклоида, принцип наименьшего действия
Аноним 31/05/18 Чтв 01:50:07  439801
2a528352dc.png (18Кб, 1182x111)
>>439791
Аноним 31/05/18 Чтв 01:51:28  439802
Что будет чувствовать человек, не чувствующий боль, опустив воду в кипяток? Просто очень тёплую воду?
Аноним 31/05/18 Чтв 03:11:55  439803
>>439802
Будет чувствовать то же, что и ты. Первое ощущение при ожоге - ебический жар, а вовсе не боль.
Аноним 31/05/18 Чтв 09:33:43  439819
temp.png (2Кб, 577x255)
>>439794
>>439801
Хуй с ним. Поясните, почему в одном случае скорось выше, чем в другом. Тип потому что глубже? К чему должен стремиться путь, для быстрейшего прохождения от точки до точки, к полуокружности? Почему?
Почему шарик в 1 сучае скатится быстрее, чем во втором? (пик)
Аноним 31/05/18 Чтв 09:34:11  439820
>>439797
Спс.
Аноним 31/05/18 Чтв 13:46:52  439838
>>439819
Есть особая кривая, называется брахистохрона. Её можно получить, пользуясь немного усложненной математикой первого курса.
Аноним 31/05/18 Чтв 14:42:18  439845
1.gif (25Кб, 488x203)
>>439819
Выше уже написали про брахистохрону. Почему по ней спуск быстрее? Ну смотри, скорость зависит от ускорения на время, а ускорение в данном случае зависит от величины уклона. Очевидно, чтобы прийти в конечную точку как можно скорее, нужно набрать как можно большую скорость как можно раньше. Шарик справа имеет постоянное ускорение, шарик слева - ускоряется намного быстрее в начале и медленнее в конце, так что шарик слева в начале пути имеет бОльшую скорость и выигрывает. Брахистохрона вообще имеет отрицательное ускорение в конце пути (т.е. там шарик замедляется), но гораздо большее ускорение в начале. Однако, если дугу выгнуть ещё больше, так чтобы в начале уклон был вообще под 90 градусов и шарик свободно падал, а затем катился по горизонтальной плоскости, то в таком случае ему приходится проходить гораздо большее расстояние и он из-за этого уже проигрывает брахистохроне (но выигрывает по сравнению с кратчайшей прямой от точки начала спуска до точки конца).
Аноним 31/05/18 Чтв 15:33:24  439848
efer.jpg (90Кб, 884x505)
То что скорость света не зависит от скорости источника мне тут пояснили. Но не пояснили почему не зависит. Как так?
Аноним 31/05/18 Чтв 15:44:32  439851
>>439848
>Как так?
Экспериментальный факт. Никто не знает почему так.
Аноним 31/05/18 Чтв 15:53:05  439852
>>439848
Потому что от скорости источника зависит геометрия пространства-времени с точки зрения этого источника.
Вот ты стоишь на месте и светишь фонариком с приложенной к нему линейкой длиной в один метр и засекаешь за какое время свет от фонарика дойдёт до конца линейки. А ещё один чувак разогнался до 0.9с и проводит тот же эксперимент - и с точки зрения того чувака свет доходит до конца линейки ровно за то же время. Но при этом если ты посмотришь на чувака, он будет выглядеть расплющенным в направлении движения, и его часы будут идти медленней. Ещё ты увидишь, что свет, который испускает его фонарик, проходит большее расстояние и за большее время, но если сверяться с часами того чувака, то время между испусканием и достижением конца его линейки будет таким же, как у тебя, потому что его время замедлилось.
Вкупе искажение пространства-времени даёт такой эффект, что, во-первых, скорость света одинакова независимо от скорости, во-вторых, невозможно определить абсолютную скорость - совершенно без разницы кто из вас летит со скоростью 0.9с - ты или тот чувак, важно что вы относительно друг друга движетесь с такой скоростью.
Аноним 31/05/18 Чтв 19:16:08  439868
Какая-нибудь организация пробовала разместить памятник с посланием для людей будущего из прочного материала на видном месте, чтобы эта штука теоретически могла очень долго простоять в этом расположении? В послание должно быть указано, что если люди будущего создали машину времени, то они должны переместить в место размещения памятника, например через час после его установки. Или в какую-то ближайшую дату и время.
Аноним 31/05/18 Чтв 19:51:54  439871
>>439868
Мы с посонами во дворе в 12 лет слелали такуй хуйню после просмотра какого-то фильма о времени. Закрутили записку в баклаху и закопали в подвале. Никто не прилетел.

>в место размещения памятника, например через час после его установки
Мы были лучше. Мы просто решили, что напишем в записке, что пидоры из будушего должны прилететь сегодня вечером. А саму записку закопали на следующий день, потому что лень было искать баклаху. И да, мы прекрасно понимали, что если не закопать, то в том что никто не прилетел может быть наш косяк.
Аноним 31/05/18 Чтв 21:23:12  439877
>>439871
Нужно памятник который будет хорошо видно, из топовых материалов, что надпись хорошо заметно в будущем, даже через множество лет. Лучше разместить на каждом материке и не один. Неужели никто не пытался провернуть?
Аноним 31/05/18 Чтв 22:36:31  439885
Нахуй нужна вакцинация от вирусов, типа гриппа, если вирусы нельзя, так сказать, вылечить? И вообще, насколько обоснована вакцинация от вирусов?
Аноним 31/05/18 Чтв 23:18:40  439887
>>439885
Узнай для начала что такое вообще вакцинация, и вопрос отпадет сам собой.
С вирусами борется иммунитет, но только с теми которые может опознать как вирус. Вакцинация - это как раз обучение иммунной системы к опознанию нового(для нее) вируса.
Аноним 01/06/18 Птн 00:56:44  439890
>>439838
>>439845
Брахистохрона это конечно круто, только вот на видео не спуск, перемещение по горизонтали(возврат на ту же высоту) а не спуск.

При чем не обязательно сама "дорога" должна быть волнистой: катаясь на велике с амортизаторами заметил что если спускаясь с горки "подпрыгивать", но не крутить педали, велик катится быстрее чем если просто нихуя не делать. То же но в меньшей мере при движении по горизонтали.
Возможно таким способом можно избавиться от трения или даже передвигаться(виброход вроде как на том же принципе основан).

Аноним 01/06/18 Птн 01:38:15  439892
Через сколько лет у людей появятся копыта?
Аноним 01/06/18 Птн 07:01:26  439897
>>439892
Поддвачну, важный. Вопрос. Ногти бесполезный рудимент на ногах, как и пальцы. На хуй они? Руки еще погимаю, там в жопе поковырять, носу. На балалайке потергать струны. А на ногах то блять? Сланцы носить. Неее, я за копыта. И обувь покупать не надо, домой зашел, молотком грязь, лед отбил и охуеть. Можно украшать как хошь, давайте ответ бляди. Хочу копыта!
Аноним 01/06/18 Птн 09:41:06  439900
DeVVG05WsAE4gC.[...].jpg (112Кб, 925x749)
>>439897
никогда, у двуногих должна быть стопа для баланса
Аноним 01/06/18 Птн 09:55:04  439901
12.jpg (300Кб, 850x587)
>>439900
Например у страуса пизже ноги чем у человека. Хотет такое.
Аноним 01/06/18 Птн 09:56:41  439902
>>439897
Ампутируй голеностоп и закажи протез в форме копыта.
Аноним 01/06/18 Птн 11:22:46  439906
>>439890
>Возможно таким способом можно избавиться от трения
Не, невозможно.
Сядь и конкретно посчитай, возьми синусоиду, дай начальную скорость. И дальше школьная задачка на скольжение.
Аноним 01/06/18 Птн 12:53:33  439912
>>439790 (OP)
как в методе ближайшего соседа находится/выбирается k? И в чем вообще суть этого метода? Как он работает?
Аноним 01/06/18 Птн 13:13:48  439917
>>439900
Стоподаун закукарекал
https://m.kp.ua/kiev/life/505900-chelovek-buduscheho-poteriaet-zuby-zato-pryobretet-kopyta
Аноним 01/06/18 Птн 14:14:11  439924
>>439890
Там есть спуск. Шарик начинает на уклоне в обоих случаях. Шарик, который катается по холмам - больше потенциальной энергии преобразует в кинетическую (проще говоря - он быстрее, потому что он ниже). Вот и всё.
Аноним 01/06/18 Птн 23:16:15  439945
Правильно ли, что во время дождя время течёт иначе, нежели в солнечную погоду, если да то почему?
Аноним 01/06/18 Птн 23:36:01  439947
56848a29b89b956[...].jpg (117Кб, 760x608)
>>439945
Аноним 01/06/18 Птн 23:52:13  439948
>>439947
Шта
Аноним 02/06/18 Суб 00:13:52  439950
>>439945
Нет. Тут могут быть разве что какие-то субъективные ощущения.
Аноним 02/06/18 Суб 00:18:34  439951
-пехота США.jpg (49Кб, 600x458)
>>439948
Ты не поймёшь.
Аноним 02/06/18 Суб 06:41:07  439957
Что будет, если около центра крупного урагана взорвать бочку ботулотоксина? Круто будет?
Аноним 02/06/18 Суб 16:06:44  439975
>>439957
Ботулотоксин распадётся в момент взрыва от высокой температуры. Обращайся ещё.
Аноним 02/06/18 Суб 16:15:06  439976
>>439975
Ну а если взрыв низкотемпературный. Или бочка хрупкая, достаточно небольшого скачка давления
Аноним 02/06/18 Суб 19:29:36  439983
Можно ли вывести физические формулы чисто математически? Могу в матанализ и гугл чуть больше, чем среднестатистический школьник. Диффурчики и интегральчики не пугают. У Сивухина в первом томе, буквально в самом начале, очень понятно и красиво выводятся основные кинематические формулы. Есть что-то похожее хотя бы по электродинамике? Где искать, что читать?
Сам я практически ничего не нашёл по этому поводу. Ну, еще в справочнике Яворского есть выводы некоторых формул вкратце, но их мало и на каждую тему слишком много ненужного текста, как по мне.
Аноним 02/06/18 Суб 21:12:19  439992
>>439983
Я же тебе уже ответил. в /un
Что тебе еще надо?)
Аноним 02/06/18 Суб 22:56:46  439995
>>439992
Я в одно и то же время написал на обе доски, потому что не знал, где мне смогут ответить. На /un абитура только и делает, что ЕГЭ да поступление обсуждает 24/7.
Спасибо за ответ.
Аноним 03/06/18 Вск 00:23:34  440004
Magnitnaya-levi[...].png (250Кб, 423x359)
мэдскилз.png (9Кб, 296x264)
>>439790 (OP)
Теоретически можно ли:

Взять ракету, присобачить к ней со всех сторон магниты и поставить ее на рельсы типа по которым идут поезда маглев (по тому же принципу) и по кругу разогнать ее до нужной скорости (понятно что неимоверно большой) чтобы она могла выйти на орбиту и выпустить ее туда пустив по уклону в верх?

Если сделать это в со всех сторон закрытой шахте и с вакуумом поможет ли это? (сопротивление воздуха будет меньше)

В чем состоит лимит скорости в технологии маглев? От чего эта скорость зависит и какой у нее теоретический предел?

Аноним 03/06/18 Вск 00:29:36  440005
>>439868
Хокинг делал подобное, но с вечеринкоц вместо памятника
Аноним 03/06/18 Вск 01:08:12  440009
Бывают ли радиоактивные катализаторы? Молекулу там расхуячить аль энергией снабдить для пущей резвости.
Аноним 03/06/18 Вск 01:51:24  440012
Аноны, нужна помощь! Кто за ночь может сделать корреляцию в spss по 3-м опросникам?
Аноним 03/06/18 Вск 03:31:10  440017
>>440004
Этого треда не существовало бы если б посты писали не сюда а в Гугл:
https://ru.wikipedia.org/wiki/%D0%9A%D0%BE%D1%81%D0%BC%D0%B8%D1%87%D0%B5%D1%81%D0%BA%D0%B0%D1%8F_%D0%BF%D1%83%D1%88%D0%BA%D0%B0

Да, такое возможно, но нужно ли оно практически. Лучше уж тогда космический лифт или орбитальный самолет запилить.

>В чем состоит лимит скорости в технологии маглев?
Трение воздуха, ты правильно догадался.
Если воздуха нет - пора подумать о релятивистских фокусах вроде ограничения в затрате энергии на ускорение.
Аноним 03/06/18 Вск 03:39:29  440018
1527983521719.gif (2418Кб, 495x525)
>>440017
https://ru.wikipedia.org/wiki/%D0%9F%D1%83%D1%81%D0%BA%D0%BE%D0%B2%D0%B0%D1%8F_%D0%BF%D0%B5%D1%82%D0%BB%D1%8F

>Пусковая петля или петля Лофстрома — опубликованный проект системы кабельного транспорта, предназначенного для вывода грузов на околоземную орбиту. В основе проекта лежит закольцованный шнур (петля), непрерывно движущийся с огромной скоростью (12—14 км/с) внутри вакуумной трубы. Чтобы шнур не соприкасался со стенками трубы, они разделены между собой магнитной подвеской, аналогично тому, как это сделано в магнитоплане. В целом это устройство представляет собой грандиозное сооружение длиной около 2000 км, а сама петля должна подниматься на высоту до 80 км и держаться на ней за счёт момента инерции вращающегося шнура. Вращение шнура по сути переносит вес всего сооружения на пару магнитных подшипников, которые его поддерживают, по одному на каждом конце.


Охренеть оказывается такое уже придумали. В любом случае спасибо анонасик. А если бы я сразу все вбивал в педовикию то что бы ты тогда делал в 3:30 ночи?
Аноним 03/06/18 Вск 03:41:19  440019
>>440018
Хз, например делил на ноль в соседнем треде.
Аноним 03/06/18 Вск 13:46:00  440046
Вопрос может больше по технике, но всё же задам тут.
Дано:
балон с газом пусть будет ацетилен или углекислый с давлением в ~25 атм 25 кг/см2, вроде в атмосферы перевёл правильно.
Камера с давлением >25 атмосфер.
Вопрос: как подавать туда газ непрерывно?
Аноним 03/06/18 Вск 16:51:23  440055
Учусь на быдлокодера, хочу в дальнейшнем, когда появятся средства, выкатиться из айти и уйти в биологию (биоинформатику либо нейросаенс), в которой я пока 0. Чтобы совместить полезное с бесполезным, думаю взять на диплом какую-нибудь софтину, связанную с будущей моей специальностью.
Есть какая-нибудь идея софтины/библиотеки для сферы биоинформатики, которую мог бы реализовать ньюфаг (с гуглом под рукой), но которой до сих пор нет, или есть но сделана коряво? Слыхал, во многих научный сферах до сих пор работают с древним и/или неоптимизированным софтом, например.
Аноним 03/06/18 Вск 20:47:41  440069
Правильно ли понимаю, что раньше мир считался строго определенным, детерминированным. Это означает, что если знать расположение всех частиц во вселенной и их направление движения, то можно со 100% вероятностью узнать прошлое и будущее в любой дискретный момент времени. Это называется демоном Лапласа. Но после открытий прошлого века в области квантовой механики, оказалось, что наблюдатель вносит изменения в наблюдаемый объект. Это убивает демона Лапласа и делает наш мир более непредсказуемым или как?
Аноним 03/06/18 Вск 20:49:01  440070
1316913991002.gif (6Кб, 569x510)
я не понимаю в каких случаях используются итерационный поиск корней уравнений, а в каких можно найти closed решение? Как это определять, и в каких случаях применение итерационных методов оправдано? Про невозможность найти общее решение для полиномов >= 5 степени я знаю.
Аноним 03/06/18 Вск 21:19:12  440080
>>440069
С одной стороны да, с другой стороны, уравнения КМ тоже все детерминированные, и если знать волновую функцию вселенной, то сможешь знать и будущее и прошлое.
О природе волновой функции и роли наблюдателя до сих пор ведутся споры.
Аноним 04/06/18 Пнд 01:29:38  440097
Почему говорится что уничтожение информации приводит к хуёвым последствиям? Ведь нигде не сказано, что физика должна предсказывать прошлое.
Аноним 04/06/18 Пнд 02:48:17  440098
Вот говорят, что вечного двигателя нет.
Но вот смотрите. вот кидаешь ты монету стремящееся к бесконечности число раз, и чем больше ты кидаешь монету, тем с меньшей вероятностью у тебя будет ровно одинаковое количество выпадений и орла и решки. то есть не равно 50 процентов. ВД же запрещают из за термодинамики, и конкретней из за энтропии которая всё же может нарушатся в астрономически малой вероятности или в малых масштабах. И может оказаться такое, что в определённых конфигурациях системы ВД всё же статистически возможен. Правильны ли мои рассуждения?
Аноним 04/06/18 Пнд 03:14:43  440100
изображение.png (152Кб, 630x354)
>>440069
Это убивает демона Лапласа и делает наш мир более непредсказуемым или как?
Демон Лапласа на то и демон что может то что нереально по определению.
В данном случае он либо не вносит изменения либо наперед знает о них, нарушая причинно-следственную связь.
Вообще, демон Лапласа больше связан с фатализмом: если будущее жестко детерминировано, то и сам демон как часть системы знает о своих дальнейших действиях и мыслях.
Аноним 04/06/18 Пнд 03:16:02  440101
>>440097
>Почему говорится что уничтожение информации приводит к хуёвым последствиям?
Где говорится? В физике нет критерия "хуевые последствия", любые последствия это просто последствия.
Аноним 04/06/18 Пнд 03:19:42  440102
>>440098
>ВД же запрещают из за термодинамики, и конкретней из за энтропии которая всё же может нарушатся в астрономически малой вероятности или в малых масштабах.
Для существования ВД 2 рода(мы сейчас о нем говорим) нужно чтоб энтропия уменьшалась, при чем постоянно а не с вероятностью, стремящуюся к нулю.
Аноним 04/06/18 Пнд 16:18:36  440129
>>440004
И да, и нет. Да в том, что сама схема принципиально возможна (только затем в высшей точке орбиты нужна будет корректировка и соответственно топливо, иначе запускаемая фигня вернётся на высоту трамплина). А нет потому, что это практически невозможно на Земле - мешает атмосфера. ОЧЕНЬ мешает. И не при разгоне, а именно при преодолении этой самой атмосферы - как только ты выпускаешь ракету из вакуумированной шахты, она на безумной скорости (минимум 8км/с для первой космической без учёта сопротивления воздуха, с учётом - где-то очень далеко, вероятно, сотни км/с) врезается в плотную атмосферу, причём направление движения не вверх, а под небольшим углом к земной поверхности, ведь нам нужно выйти на орбиту, а не просто подпрыгнуть и упасть обратно. При такой скорости никакие конструкционные материалы и никакая форма не спасут от сгорания в атмосфере - обшивка почти сразу же раскалится на несколько тысяч градусов и начнёт плавится со скоростью вплоть до сантиметров в секунду.
Строительство же выходного конца шахты на десятки километров в высоту и тысячи километров шириной - затея даже более безумная, чем космический лифт.
Аноним 04/06/18 Пнд 16:24:15  440130
>>440018
А вот это интересная идея, намного лучше обычного трамплина. Странно, что я не находил её раньше.
Аноним 04/06/18 Пнд 16:28:29  440131
>>440129
> затея даже более безумная
А что если очень длинный дирижабль на водороде, и на него облегченный вариант пушки - все это в разряженной атмосфере может и проканать.
Аноним 04/06/18 Пнд 20:08:36  440136
thermostat.jpg (47Кб, 500x367)
Котаны ученые, посоветуйте бюджетное оборудование для домашней биохимической лабы. В первую очередь интересует какой-нибудь дешевый термостат-инкубатор.
Можно ли использовать какой-нибудь металлический шкафчик с термостатом для регуляции температуры и нагревом лампочкой накаливания?
Можно ли доверять китайцам при покупке расходников (агар, чашечки и т.п)?
Автоклавировать буду в мультиварке, лол.

Если есть какие-то гайды от суровых DIY-умельцев, то подкиньте.
Аноним 04/06/18 Пнд 20:28:40  440137
>>440136
>нагревом лампочкой накаливания
>Автоклавировать буду в мультиварке
Проиграл.

>дешевый термостат-инкубатор
Берешь ТЭН от печки, нужных размеров ящик(если небольшой то сойдет и сама печка), если это растения уф-лампу, Ардуину, термодатчик, и собираешь из всего этого инкубатор.

Аноним 04/06/18 Пнд 23:14:41  440144
Есть ли хоть один случай, подтверждающий легенду о том, что в тёмном помещении с большим количеством растений можно задохнуться? Про то, что растения без света поглощают кислород, мне известно. Гугл ничего определённого не выдал.
05/06/18 Втр 06:56:09  440148
Когда автоматизации станет больше?
Ведь это счастье полные тред с ботами, которые ведут с тобой диалог.
Нет всяких людей в услугах, никто не наёбывает каждую минуту.
Что прочитать про это?
Аноним 05/06/18 Втр 08:42:19  440152
>>440144
Откуда ты взял эту легенду? В любом помещении можно задохнуться, если в нём нет достаточной вентиляции. Вот это труъ сториъ.
Аноним 05/06/18 Втр 11:32:33  440156
>>440148
>полные тред с ботами, которые ведут с тобой диалог.
/po/
>Нет всяких людей в услугах, никто не наёбывает каждую минуту.
В таком случае и услуги-то не особо нужны, большинство услуг направлено как раз на наеб и получение прибыли.
Да и человеческий труд банально дешевле чем автоматизация. Представь тебе нужно создать робота-дворника или робота-солдата - он никогда не окупится, ведь таких роботов нужно дохуя и они будут быстро ломаться. Другое дело твой любимый народ, готовый за еду горбатиться весь день.


Аноним 05/06/18 Втр 11:46:47  440157
>>440137
>Автоклавировать буду в мультиварке
>Проиграл
Норм, чо. Главное, чтобы мамка не спалила.
Аноним 05/06/18 Втр 12:19:21  440158
Эм излучение является первичным поражающим фактором ядерного взрыва? А уже потом из-за нагрева воздух расширяется в ударную волну? Т.е. если сделать корпус ядерки из прозрачного нерушимого стекла, примерно так же ёбнет?
Аноним 05/06/18 Втр 13:33:56  440162
>>440158
Нет, конечно. Если ты оставишь только электромагнитное излучение - пиздец тоже будет, но совсем не настолько разрушительный и взрывной волны ты не получишь. Нейтроны, альфа частицы, тяжёлые ядра - именно эти лучи радости в первые моменты взрыва формируют фронт взрывной волны, а ЭМ-излучение только подготавливает почву - ионизирует частицы.
Аноним 05/06/18 Втр 13:51:48  440163
>>440162
Ну под эм я имею в виду тепловое излучение, нееобически всё нагревающее. У физички плакат висел, что в эпицентре 10^10 температура, как бы это дохуя.
Аноним 05/06/18 Втр 14:04:14  440164
Есть два тупых вопроса:
1. Насколько реальны и изучаются случаи, когда внутри человека прорастало растение. Этот детский страх проглотить косточку от овоща/фрукта и страшилки от взрослых, что она прорастёт внутри получается имеет основания? Я читал в каких-то жёлтых газетах о иголках, прорастающих на коже, стеблях бамбука, растущих в лёгких и прочих заболеваний (лол). Так вот, это исследовалось учёными, проводились ли опыты в лабораторных условиях по скрещиванию растения и животного? Как такое искать? Есть шанс прорасти у косточки или у вдохнутой пыльцы?
2. Мы живём в 3D-измерении или время это не процесс, а тоже dimension? Может ли житель nD-измерения узнать как выглядит или предсказать существование (n+1)D измерения? А n-1D? Ведь мы не можем найти в нашем мире ни 2D, ни 1D существ. Так может измерение всего одно, а остальное это просто математическая абстракция? Или всё же есть 4D, например такое, в котором все люди являются одной особью? Помогите понять, а то уже сплю плохо от таких мыслей.
Аноним 05/06/18 Втр 14:06:14  440165
>>440163
Если ты только тепловое имел в виду, то энергия ещё меньше. ЭМ-излучение от ядерного взрыва включает в себя весь спектр - от радио, до гаммы.
Аноним 05/06/18 Втр 14:30:40  440167
1.gif (4135Кб, 655x374)
>>440164
1) При наличии иммунитета практически нереально. При синдроме иммунодефицита - могут закрепиться некоторые виды грибов. Остальным растениям в теле охуенно неуютно. В желудке - вообще без шансов. В лёгких - нереалистично, т.к. попасть незаметно могут только споры, но у них нет запаса веществ для прорастания, а в самих лёгких необходимых веществ не найти. А крупные косточки с запасом веществ вдохнуть незаметно не получится.
Пыльца вообще не прорастает, пыльца - это считай то же, что сперматозоиды у животных.
2) Мы в четырёхмерном пространстве живём. И пространства с размерностью >4 мы вполне можем представить хотя бы математически. А вот представить себе в мозгах - тут немного сложнее. Чтобы что-то представить, надо с этим сталкиваться и этому обучаться. Но наша вселенная трёхмерная, так что нам с пространством большей размерности сталкиваться не приходится. Но если бы мы могли, например, создать сильный ИИ, то никакой закон природы не запрещал бы обучать его ориентированию одновременно и в трёхмерном, и в четырёхмерном пространстве. Найти 2D или тем более 1D существ мы не можем, потому что такая конфигурация практически запрещает формирование жизни - в нём крайне ограничена полимеризация и транспорт веществ. А в четырёхмерном пространстве с ещё одним временным измерением невозможно существование стабильных орбит небесных тел, так что там будет практически невозможен нуклеосинтез - потому что не смогут сформироваться даже звёзды.
Кстати, ты и так умеешь представлять себе четырёхмерное пространство, правда скорее в конфигурации 3.5D. Представь себе человека, который идёт по аллее - и это вот оно. Полная конфигурация - это весь путь от начала и до конца одновременно, как будто бы червь, который на срезе будет иметь форму человека. Собственно, ты как раз и представляешь срез этой конфигурации в виде трёхмерного пространства и как бы "сканируешь" полную конфигурацию её, перемещая этот срез вдоль оси времени.
Если бы мы жили в двухмерной плоскости и пытались представить себе третье измерение, то это было бы возможно так же - как двумерный срез трёхмерного объекта, проходящего через плоскость. Смотри картинку.
Аноним 05/06/18 Втр 14:34:14  440168
1.gif (4011Кб, 462x329)
>>440167
>>440164
Она же, но с самим объектом.
Аноним 05/06/18 Втр 14:42:12  440169
>>440164
> Мы живём в 3D-измерении или время это не процесс, а тоже dimension?
У нас все определения времени завязаны на пространстве. А определений пространства нет, их мы может только завязать на времени. Поэтому тут вопрос скорее не в том, время это еще одна координата? А в том. а пространство это что? Иллюзия ли?
Аноним 05/06/18 Втр 16:25:49  440175
bbcHowToGrowAPl[...].webm (8949Кб, 360x200, 00:02:15)
>>440144
Мне известно только про опыт Пристли с двумя стульями емкостями, в одну он сажал мышь точеную и растение, а в другую просто мышь. В итоге мышь, которая без растения, умирала сразу, а с растением жила ещё недельку. Но дело было в том, что Пристли обнаружил только благотворное влияние растений на воздух и не заметил что это все происходит днем, поэтому когда у некоторых ученых не получилось воспроизвести опыт, Пристли ещё несколько лет искал в чём причина. За его опытами и последующим открытием важности солнечных лучей для фотосинтеза подсмотрел Ингенхауз и выпустил на год раньше книгу с описанием, от чего Пристли словил батхерт. И ещё BBC ставила подобный опыт в фильме How To Grow Planet, первый эпизод Life From Light, только с человеком (professor Iain Stewart), там они сразу включили свет, иначе он бы упал в обморок через сутки, наверно это то, что тебе нужно.
Аноним 06/06/18 Срд 01:00:53  440194
В опыте Юнга (который с двумя щелями) под наблюдателем подразумевается фиксирующий прибор или человек тоже? То есть если человек просто будет смотреть на всю эту ебалу, своими глазами, не видя этих самых фотонов, так как они слишком малы, они будут вести себя как частицы?
Аноним 06/06/18 Срд 02:10:52  440197
>>440194
Хоть это и не совсем ответ на вопрос, но вообще-то человек вполне способен наблюдать единичные фотоны.
https://www.nature.com/articles/ncomms12172
Аноним 06/06/18 Срд 02:28:13  440198
>>440158
> А уже потом из-за нагрева воздух расширяется в ударную волну?
Верно.
The high temperatures and radiation cause gas to move outward radially in a thin, dense shell called "the hydrodynamic front". The front acts like a piston that pushes against and compresses the surrounding medium to make a spherically expanding shock wave.

Для воздушного взрыва:
Blast—40–50% of total energy
Thermal radiation—30–50% of total energy
Ionizing radiation—5% of total energy (more in a neutron bomb)
Residual radiation—5–10% of total energy with the mass of the explosion

Если предположить, что можно как-то изолировать ударную волну, то у нас уже половина энергии отсекается. Тут, скорее, все зависит от того, какого размера твой неразрушимый корпус. Чем он больше, тем меньше энергии придется на ударную волну.

А вообще, абсолютная неразрушимость и прочие имба-условия - это как-то несерьезно. Особенно когда мы имеем дело с эпицентром ядерного взрыва.
Аноним 06/06/18 Срд 02:57:01  440201
>>440197
>https://www.nature.com/articles/ncomms12172
Ну так даже лучше. Ну так человек, наблюдающий за происходящим без прибора считается за наблюдателя? Если так то тут уже не скинешь вину за странное поведение частиц на прибор, потому как никакого воздействия на них оказываться не будет.
Аноним 06/06/18 Срд 03:19:38  440203
>>440201
Наблюдение, читай взаимодействие. Причем не любое, а именно узнавание нужной величины. Узнавая импульс по оси y, координату по оси x спокойно можешь измерять, без всяких проблем.
Аноним 06/06/18 Срд 03:46:12  440205
>>440198
> А вообще, абсолютная неразрушимость и прочие имба-условия - это как-то несерьезно. Особенно когда мы имеем дело с эпицентром ядерного взрыва.
Это для простоты, физики же тоже абстрактные модели делают, но с хитровыебанной формулировкой, всякие там двумерные вакуумы, эфективная отрицательная температура и прочие кони. Я в такое не могу, начали бы задавать сотни уточняющих вопросов, в простую суть не вникли и забили когда уже в сослагательном наклонении будет допустимо употреблять "бы" один раз?.
Сказ о физикс Аноним 06/06/18 Срд 06:12:23  440208
Сегодня я сидел и думал. Думал о звороте веществ в Натурии.
Так вот. Раньше, 60 млн лет назад, динозавров развелось видимо-невидимо. Масса биологических существ на поверхности была огромна, в том числе растениеподобных существ.
Корч, они умерли в течение миллионов лет и создали к нынче нефть. Есть это полностью сгнившие звери-рептилии.
Сегодня их нефти делают белок, кормят им человеков и толстых женщин.
Круговорот на лицо.
Аноним 06/06/18 Срд 08:21:44  440213
>>440208
Ты долбоеб, нефть не из динозавров образовалась.
Аноним 06/06/18 Срд 10:40:18  440217
>>440203
То есть если человек будет стоять и смотреть на всю эту ебалу непосредственно фотоны будут вести себя как волны?
Аноним 06/06/18 Срд 13:30:32  440228
>>440213
а из чего тогда?
Аноним 06/06/18 Срд 13:51:13  440229
Есть ли жизнь в естественных нефтяных резервуарах? Чем не питательная среда же.
Аноним 06/06/18 Срд 15:29:58  440231
>>440217
При чём тут человек то? Человек фотоны не видит, пока они ему в глаз не прилетят, а значит и не взаимодействует с ними.
Аноним 06/06/18 Срд 16:48:33  440234
>>440231
А в чем тогда магия эксперимента? Получается же измерительные приборы всё портят
Аноним 06/06/18 Срд 17:50:58  440235
>>440234
>магия
Вут? При чем тут магия?
>Получается же измерительные приборы всё портят
Ну смотри, мы можем измерить, а потом сразу информацию стереть. При этом ничего не попортим.
Аноним 06/06/18 Срд 18:00:59  440236
>>440235
>Вут? При чем тут магия?
это образно
>Ну смотри, мы можем измерить, а потом сразу информацию стереть. При этом ничего не попортим.
так вроде частица меняла поведение от факта наблюдения (или измерения, я не знаю чем они в физике отличаются), а не от того, получит ли информацию наблюдатель или нет
Аноним 06/06/18 Срд 18:02:41  440237
>>440236
Ну видишь, твоя версия противоречит эксперименту.
Аноним 06/06/18 Срд 18:07:37  440239
>>440237
то есть я неправильно понял эксперимент?
Аноним 06/06/18 Срд 18:11:38  440240
>>440239
Посмотри большее количество разных.
А лучше изучи еще два.
1) С квантовым ластиком
2) С отложенным выбором
Аноним 06/06/18 Срд 18:24:52  440243
>>440240
ну про отложенный выбор читал, а вот про ластик немного недопонял. то есть если они фиксируют картину, они не наблюдают интерференции, а если они фиксируют, но не смотрят на результаты, то интерференция будет?
Аноним 06/06/18 Срд 18:54:35  440246
>>440243
> но не смотрят на результаты
Они не могут посмотреть на результаты дырок.
Аноним 06/06/18 Срд 18:56:38  440247
>>440246
не понел, объяснишь?
Аноним 06/06/18 Срд 18:58:47  440248
>>440247
Детектор считывает, через какую дырку прошла частица, но уничтожает информацию.
Аноним 06/06/18 Срд 19:00:06  440249
>>440248
и интерференция наблюдается в таком случае?
Аноним 06/06/18 Срд 19:09:18  440252
>>440249
Да
Аноним 06/06/18 Срд 19:12:31  440255
>>440252
бля, это сложно, получается все таки детектор не влияет, а влияет наблюдатель, и как это объяснить, мм?
Аноним 06/06/18 Срд 19:18:06  440256
>>440255
>и как это объяснить, мм?
Есть несколько способов. Но все упираются в свойства уравнения Шредингера.
Аноним 06/06/18 Срд 19:18:54  440257
>>440256
можешь объяснить плебсу? плебс это йаа
Аноним 06/06/18 Срд 19:25:27  440259
>>440257
Бери любую интерпретацию КМ и смотри как она это объясняет. Конечно проще всего это делает многомировая.
https://youtu.be/toryl7BkNM0
Аноним 06/06/18 Срд 19:32:28  440260
>>440259
а теория струнн?
Аноним 06/06/18 Срд 19:42:02  440261
>>440260
При чём тут теория струн?
Аноним 06/06/18 Срд 19:43:08  440262
>>440261
ну она же вроде пытается соединить классическую и квантовую физику
Аноним 06/06/18 Срд 20:16:47  440263
>>440262
Классическая физика с самого начала часть квантовой, там ничего соединять не нужно.
Она пыталась соединить ОТО с квантовой. Но хуево получилось. Все забили хер вроде уже.
Аноним 06/06/18 Срд 20:18:37  440264
>>440263
на теорию струн? она же типо самая перспективная щас, не?
Аноним 06/06/18 Срд 20:31:41  440265
>>440264
Лет 20 назад может и была. Сейчас вроде нет.
Аноним 06/06/18 Срд 21:02:30  440266
>>440194
>повлиять на эксперимент с фотоном
>наблюдением
>когда он въебался нахуй ему в глаз и исчез
кек
хмм, даже не знаю, повлияет ли это на эксперимент или нет...
Аноним 06/06/18 Срд 21:20:53  440268
>>440208
толсто
Аноним 06/06/18 Срд 21:34:56  440269
>>440228
Из неорганики
Аноним 07/06/18 Чтв 10:13:17  440300
Вкачусь с тупым вопросом в вашу беседу о корпускулярно волновом дуализме этом.
Насколько велика вероятность того, что фотон это и не частица, и не волна, а что то эдакое третье, чьи проявления мы видим то как волну, то как частицу?
Аноним 07/06/18 Чтв 10:51:35  440301
Фраза: "Я занимаюсь этим делом час. Могу ли я заниматься этим делом больше/меньше времени?"

Есть ли научный термин, чтобы написать вот это "больше/меньше" одним словом в таком контексте?
Аноним 07/06/18 Чтв 11:25:51  440302
>>440300
>Насколько велика вероятность того, что фотон это и не частица, и не волна, а что то эдакое третье

100%
Аноним 07/06/18 Чтв 11:37:03  440303
>>440300
Почему-то многих смущают волны, но никогда не смущало описание частиц как БЕЗРАЗМЕРНЫХ ТОЧЕК с БЕСКОНЕЧНОЙ ПЛОТНОСТЬЮ
Аноним 07/06/18 Чтв 12:27:37  440305
>>440302

> 100%
Почему же тогда нет внятной и понятной модели и теорий?
Аноним 07/06/18 Чтв 12:29:39  440306
>>440303
Это действительно так и описывается?
Аноним 07/06/18 Чтв 12:36:23  440307
>>440305
Внятной для кого?
Аноним 07/06/18 Чтв 12:48:37  440309
>>440307
Внятной для задающего тупые вопросы, конечно. Серьезно, я читал научпоп, и что то нигде такой четко прописаной теории того, что дуализм фотона следствие того, что он и не волна и не частица, не увидел
Аноним 07/06/18 Чтв 12:52:11  440311
>>440301
Δ
>>440309
Представляю как ты охуеешь со стерильного нейтрино, которое вообще не взаимодействует с веществом https://www.livescience.com/62721-sterile-neutrino-detected-fermilab.html
Аноним 07/06/18 Чтв 12:57:39  440312
>>440309
Дуализм, это старая очень старая байка. Которую таскают уже хрен знает сколько лет.
Я просто не могу понять, что ты хочешь под "четко прописанной"? Математика строгая? Ну так квантовая электродинамика. и квантовая хромодинамика строгие тогда.
Под "внятной" ты наверное имеешь ввиду что-то представимое? Прости, но мы обезьяны, и нашу воображалку создавала эволюция чтобы бегать от тигров, а для этого достаточно три измерения уметь представлять. Элементарные частицы там не укладываются.
Аноним 07/06/18 Чтв 13:18:02  440313
>>439790 (OP)
Я тупой, извините, но у меня вопрос.
Он отчасти научный, отчасти маняфантазийный.
Остановка времени в фильмах на примере разных Флешей/Докторов Стренжей/хуйнянаме.
Имеется в виду не воздействие на разум отдельных индивидуумов, а полная остановка времени и перемещение героев в это время.
Фактически, всё пространство вокруг нас заполнено материей. И всякая материя имеет энергию. Кинетическую, потенциальную, дохуя всего.
Так вот, чем по факту чревато перемещение объекта во время полной остановки времени?
То есть, как я понимаю, объект перемещается с бесконечно большой скоростью?
Относительно законов физики, это ведь должно привести к разрушению как самого объекта, так и вообще всего, с чем он соприкоснется?
Типа, скажем, если мы переместим стакан с водой за бесконечно малый ну, в идеале, нулевой промежуток времени на условный метр, то появившееся энергия банально распидорасит всё вокруг?
Почти уверен, что в этом разделе меня на хуй пошлют, но я заебался с этими размышлениями.
Аноним 07/06/18 Чтв 13:22:07  440314
>>440313
В реальной физике вообще ограничения на скорость есть. И быстрее света не побегаешь. И да, ты разъебешь планету таким стаканом.
Но как я понимаю. В этих фильмах будто бы создается еще одна ось времени, и они двигаются в ней, захватывая всю мешающую материю вместе с собой в неё.
Аноним 07/06/18 Чтв 13:22:25  440315
Господа, объясните пожалуйста почему солнце (по фотографиям из космоса) обладает каким-то грязно-оранжевым цветом? У нашей звезды ведь максимум спектра излучения приходится где-то на 550нм, оно же должно быть зеленым.
Аноним 07/06/18 Чтв 13:26:12  440316
>>440315
Там довольно длинный хвост, и максимум не такой уж резкий. Поэтому не очень удивительно что солнце такое желтое.
>грязно-оранжевым
Играют с красками или в спектре водорода смотрят. Надо знающих анонов спрашивать.
Аноним 07/06/18 Чтв 13:56:37  440318
>>440313
Скорее всего как двигатель альбукерке. Не герой двигается сам, а герой двигает реальность с нужной ему скоростью.
Аноним 07/06/18 Чтв 13:58:51  440319
По сравнению с каким объектом в нашей вселенной земля будет иметь размер электрона?
Аноним 07/06/18 Чтв 14:14:03  440320
>>440319
С электроном, если его сравнивать в с землёй.
Аноним 07/06/18 Чтв 15:33:35  440328
>>440312
> Дуализм, это старая очень старая байка. Которую таскают уже хрен знает сколько лет.
Впервые об этом читаю. Ну пусть байка, допустим. Зачем тогда байка взята за печку, от которой начинается танец с бубном по поводу построения моделей и теорий?
> Я просто не могу понять, что ты хочешь под "четко прописанной"? Математика строгая? Ну так квантовая электродинамика. и квантовая хромодинамика строгие тогда.
Теория должна быть доступной для такого обьебоса как я, например. Прочитал и понял. Что не понял, что прочитал.
> Под "внятной" ты наверное имеешь ввиду что-то представимое? Прости, но мы обезьяны, и нашу воображалку создавала эволюция чтобы бегать от тигров, а для этого достаточно три измерения уметь представлять. Элементарные частицы там не укладываются.
Не в этом дело. Почему физики сразу не говорят, что мол вот фотон. Почему он так себя ведет, мы не знаем, сознаемся честно, бейте нас ногами. Зачем и почему принято такое допущение, что ни частица, ни волна, значит это что то третье, а не то и другое, это похоже на сказ о троице, единой но разной. Любой поп скажет, что нужно просто поверить и ниибет. Так и с дуализмом.
Аноним 07/06/18 Чтв 15:34:44  440329
>>440313
Начнём с того, что мир почернеет. До абсолютной темноты. Фотоны замерли в воздухе, электроны в проводах, так что наш герой, движимый вопреки стреле времени на своей волне даже просто не сможет найти стакан. Но это наименьшая его проблема. Вторая проблема - даже если он сможет двигаться, то каждая молекула в воздухе будет обладать бесконечной энергией - ведь относительно неё наш герой двигается со сверхсветовой скоростью. Поэтому в первый же миг после разморозки времени вселенная начнёт... делать совершенно неизвестные, но явно фатальные для всего окружающего вещи. У нас просто нет теорий, которые могли бы описывать что произойдёт при бесконечной энергии в конечном пространстве. Может быть, мир начнёт переходить к более стабильному состоянию поля Хаббла, может ещё что-то похуже.
Да, в общем, герой не только ничего не увидит, и не только не сможет поднять стакан, он не сможет вдохнуть или даже пошевелиться - любое взаимодействие с объектами в остановленном времени приведёт к тотальному пиздецу вселенной.
Аноним 07/06/18 Чтв 15:41:29  440331
>>440313
А что такое время?
Аноним 07/06/18 Чтв 16:33:44  440332
>>440328
>Теория должна быть доступной для такого обьебоса как я, например.
Схуяли? Пиздуй учиться, сразу все станет понятно. Для начала окончи школу не на двойки, потом вузик с нормальным физфаком, потом аспирантурку. Тогда и возвращайся права качать.
Аноним 07/06/18 Чтв 16:41:51  440333
>>440332
>Схуяли?
Ему так сказал на ютубе какой-нибудь популяризатор околонауки, насмотревшийся документалок.
Аноним 07/06/18 Чтв 18:11:25  440337
>>440332
> Схуяли? Пиздуй учиться, сразу все станет понятно. Для начала окончи школу не на двойки, потом вузик с нормальным физфаком, потом аспирантурку. Тогда и возвращайся права качать.
Спасибо за гайд.
Это тред тупых ответов, я понял.
Аноним 07/06/18 Чтв 18:16:33  440339
>>440333
Вы сможете ответить или мое скудоумие только способны обсуждать? Удивительно, чего я еще тут думал найти, на борде, полной долбоебов. Спасибо за ламповое общение о науке, уроды прыщавые. Научили.
Аноним 07/06/18 Чтв 18:25:13  440340
>>440339
>Вы сможете ответить
>Почему же тогда нет внятной и понятной модели и теорий?
Внятная и понятная модель и теория есть. Называется квантовая мезханика.
Аноним 07/06/18 Чтв 19:14:38  440341
>>440328
Потому что в этом и дело, не должна быть она доступной. Она привязана к жизни. А это уже минус 100 к доступности.
Это не допущение если что. И да, это просто волна, все частицы просто волны. Частицы они чисто субъективно.
Аноним 07/06/18 Чтв 20:15:00  440346
свет это электромагнитная волна
электричество в проводах это тоже электромагнитная волна?
почему тогда они такие разные и разные ли?
Аноним 07/06/18 Чтв 20:48:18  440349
Читаю конспект и не могу понять, что написал:
1) "неоднородного взаимодействующего электронного газа",
2) "неоднородно взаимодействующего электронного газа".

Понятие "неоднородное взаимодействие" никогда не слышал, но гугл намекает, что оно вроде как есть само по себе.
На 99% уверен, что 1-ый вариант корректный.

Контекст: "все свойства такого электронного газа могут быть описаны с помощью функционала от электронной плотности". Речь про теорию функционала эл. плотности.

Я тупой, вопрос тупой, etc. Но все-таки?
Аноним 07/06/18 Чтв 20:52:04  440350
>>440349
>конспект
ммм, пидорашье образование
Аноним 07/06/18 Чтв 21:17:56  440351
>>440349
вопрос снят, кек
Аноним 07/06/18 Чтв 22:40:18  440353
elec.jpg (123Кб, 924x627)
собственно пикрелейтед. объясните откуда взялись q'=R/a и a'=R^2/a
Почему R^2/a? R/a? Почему мы строим изображение q' именно там?????
Аноним 07/06/18 Чтв 22:43:10  440354
>>440341
> Это не допущение если что. И да, это просто волна, все частицы просто волны. Частицы они чисто субъективно.
Восприятие всегда субьективно, априори. Вследствие чего и вопрос об этом, очевидно из разряда тупых, что проявления дуализма фотона для неспециалиста выглядит некоректным допущением. Именно потому как описаны его проявления, складываются ощущения того, что физики пытаются описать запах жареной картошки при помощи классической музыки.
Аноним 07/06/18 Чтв 22:45:36  440355
>>440346
>лектричество в проводах это тоже электромагнитная волна?
>почему тогда они такие разные и разные ли?
в проводах электрончики
Аноним 07/06/18 Чтв 22:54:55  440357
>>440331
Дельта энтропии.
Аноним 07/06/18 Чтв 23:43:12  440358
>>440357
Кривая ануса- тоже так же красиво и загадочно читается
Аноним 07/06/18 Чтв 23:57:47  440360
>>440355
От них же вроде толку хуй да нихуя.
Аноним 08/06/18 Птн 00:23:37  440362
>>440360
с чего ты так решил
Аноним 08/06/18 Птн 00:26:09  440363
сейчас читаю книги по эволюции и появился вопрос. ну в общем известно, что сапиенсы победили неандертальцев и они вымерли. так вот вопрос, могли ли некоторые неандертальцы влиться в общество сапиенсов? ну то есть смешение между сапиенсами и неандертальцами было, насколько известно антропологам, но возможно ли, что сейчас остались предки неандертальцев где то? как это противоречит известным фактам по вытеснению сапиенсами неандертальцев?
Аноним 08/06/18 Птн 00:31:08  440365
>>440363
У людей вроде существует часть генов неандертальцев. Смешение точно произошло. Пруфы нужно поискать, я не уверен.
Аноним 08/06/18 Птн 00:34:21  440366
>>440363
Загугли кариотип неандертальцев. Если не 2n=23, тогда толку от смешения не было бы, даже если оно было бы.
Если кариотип такой же как у человека, то скорее всего я хуй знает
Аноним 08/06/18 Птн 00:38:05  440367
>>440365
ну это впринципе естественно, потому что они вытеснялись постепенно. меня интересует, могли ли остаться прямые родственники неандертальцев, или бы их давно обнаружили? просто у них там форма мозга другая, и голова поэтому тоже, но на счет головы не точно
>>440366
я не понял что ты написал, сорян. я не биолог, я в философию вкатываться пытаюсь
Аноним 08/06/18 Птн 00:39:20  440368
>>440367
>прямые родственники
У нас с тобой общие прабабки были лет 500 назад.
О чем ты?
Аноним 08/06/18 Птн 00:44:42  440369
>>440368
ну может неандертальцы еще остались? только эволюционировавшие? или они не отличались бы от сапиенсов?
Аноним 08/06/18 Птн 00:46:14  440370
p01h9j56.jpg (33Кб, 640x360)
>>440328
>Теория должна быть доступной для такого обьебоса как я
кто же тебе такое сказал то?
Аноним 08/06/18 Птн 00:52:33  440371
>>440360
ну вот смотри, если в проводе эл.маг.волны, то просто берешь обматываешь палец фольгой и суешь в розетку в дырку где фаза, ничего не должно произойти, поскольку ты экранировал палец от эл. маг. излучения.
Аноним 08/06/18 Птн 01:12:20  440372
>>440371
снимаю шляпу
Аноним 08/06/18 Птн 01:13:53  440373
drcscr.gif (191Кб, 220x220)
>>440328
>Почему физики сразу не говорят, что мол вот фотон.
Ну тебе формулу напишут и скажут - вот это фотон, и будут абсолютно правы, а если ты не можешь понять физический смысл написанного, то уж сорри.

>Зачем и почему принято такое допущение, что ни частица, ни волна
Это не допущение, это факт экспериментальный. Дуализм означает лишь то, что одни и те-же результаты могут быть получены двумя казалось бы разными путями. Это просто термин для начинающих.

>Любой поп скажет, что нужно просто поверить
Поп не имеет фактической экспериментальной базы в основе своих верований, в отличии от физики. Все сравнения с религией от тупости и невежества.
И вообще чтобы что-то понять, нужно хотя-бы начать обмазываться вменяемым научпопом.
Аноним 08/06/18 Птн 01:45:52  440374
>>440353
Тебе нужно уложиться во все условия, а самое главное условие, у тебя в проводнике потенциал должен быть нулевой.
Аноним 08/06/18 Птн 01:55:01  440375
В чем преимущества вычисления погрешности по частному случаю лучше общего?
Аноним 08/06/18 Птн 01:55:23  440376
В чем преимущества вычисления погрешности по частному случаю лучше общего?
Аноним 08/06/18 Птн 02:25:01  440378
>>440353
https://ru.coursera.org/learn/electrostatica/lecture/CZhB7/4-mietod-izobrazhienii-zaziemliennaia-i-niezaziemliennaia-provodiashchaia-sfiera
Тут всё есть.
Аноним 08/06/18 Птн 03:42:15  440384
>>439790 (OP)
Почему полевой транзистор разрушается при <200°С, в то время как для биполярного эта температура 300°С а то и 400°С?
Судя по внутреннему строению должно быть наоборот - биполярный имеет тонкий слой-базу, которая уничтожается тупо диффузией, полевик же это по сути 2 диода с кондером посередине.
Аноним 08/06/18 Птн 07:47:31  440395
>>440362
Они ж движутся там дохуя медленно, пару метров в минуту, и всё передаётся, со скоростью света или около того, по проводам полем
Аноним 08/06/18 Птн 09:56:20  440403
Если учёным дать целую днк неизвестного организма но принадлежащего к известным царствам, без инопланетных приблуд, смогут ли они реконструировать его облик и образ жизни? Насколько точно классифицируют (д семейства/рода/вида)? Особенно если организм сложный, какое-нибудь жывотнайе? Могут ли получиться неопределённые результаты? Например, этот участок указывает на корову, другой на крокодила, а тот вообще на ананас.
Аноним 08/06/18 Птн 11:01:04  440408
>>440395
ты серьезно думаешь, что от начала проводника просто летят первые электроны, а все остальные этого не замечают?
Аноним 08/06/18 Птн 11:47:47  440409
>>439790 (OP)
А есть какие-нибудь серьёзные исследования, связанные с психологией отдельных этносов или их психогенетических особенностей? На эту тему мне много попадалось всякого наци мусора, поэтому прошу вас подкинуть годноты.
Аноним 08/06/18 Птн 14:03:56  440418
>>440370
Я сам и говорю. С моей точки зрения, научпоп для гуманитариев должен да да, никто, никому, нихуя быть доступным. Почему? Очевидно же. Следовательно, теории квантовой физики хоть и не могут быть понятными от и до, однако некоторая какбе недосказанность в них есть.
Аноним 08/06/18 Птн 14:13:52  440422
>>440418
Ну смотри, книга по квантовой механике, берет одно уравнение, и выводит из него охуенно много свойств, всяких фишечек, довольно неочевидных вещей связанных с математикой и т.п. страниц 400
Вопрос, как ничего не упустить, чтобы как-то это рассказать?
Аноним 08/06/18 Птн 15:38:53  440427
>>440371
Чот он не отвечает, я уже беспокоюсь за него.
Аноним 08/06/18 Птн 15:50:53  440428
>>440403
Они могут установить его принадлежность к той или иной ветви на филогенетическом древе и ближайшего родственника с известным геномом (который фактически может оказаться разделённым на поллярда лет, потому что разные ветви древа жизни изучены в разной мере). И исходя уже ТОЛЬКО из этого восстановить его облик, как нечто подобное на ближайших родственников. По ДНК восстановить облик мы не можем (вполне вероятно даже никогда не сможем, потому что нужен не только геном, но и протеом - а это эпигенетическая наследственность).
Неопределённые результаты не просто могут, они всегда получаются, потому что в большинстве случаев источник ДНК чем-то загрязнён и просто используют то, что у эукариот (все животные и растения) ДНК состоит из небольшого числа очень длинных молекул, так что берут много клеток, перемалывают их в кашу, вычищают из неё фрагменты ДНК, сканируют их и дальше собирают как паззл до целых молекул. Если фрагмент чужероден и не принадлежит исследуемому организму, это видно по статистике встречаемости определённых участков молекулы - т.е. материала исследуемого образца как правило сильно больше и все его хромосомы более-менее равновероятно встречаются, а загрязнения всё-таки в меньшинстве и даже если из загрязнений собирается целая молекула, она встречается с сильно меньшей вероятностью и её отбрасывают.
Совсем уж неопределённые результаты получаются, когда материала очень мало и из него не удаётся собрать целые молекулы, или молекулы собираются, но с очень небольшим числом повторений - 1-2 раза. В таком случае как раз загрязнения сложно отфильтровать и такой геном считается черновым.
Аноним 08/06/18 Птн 15:53:00  440429
>>440418
Только забыл, что научпоп≠наука. И почему то распространяешь на науку то, что свойственно научпопу и считаешь что вся наука должна быть такой. Не надо так.
Аноним 08/06/18 Птн 16:25:50  440430
Можно ли сделать такую, линзу, чтобы она в дону сторону увеличивала, а в другую уменьшала?

Или это можно проделать только с системой линз?
Аноним 08/06/18 Птн 17:19:01  440432
>>440409
бамп вопросу
Аноним 08/06/18 Птн 18:02:21  440433
>>440408
Т.е. они на всей длине начинают двигаться одновременно?(с разностью около L/c)?
И их же заставляет двигаться не воздействие електронов которые ближе к началу, а таки поле?

>>440427
>Чот он не отвечает, я уже беспокоюсь за него.
как мило;3
Аноним 08/06/18 Птн 19:27:52  440435
>>440373

Если начать рассуждать о том, что формулы придуманы физиками для самообмана, это будет жирным фричеством, правда? Я понимаю, что математика является единственным способом, годным для описания мира.

> Это не допущение, это факт экспериментальный. Дуализм означает лишь то, что одни и те-же результаты могут быть получены двумя казалось бы разными путями. Это просто термин для начинающих.
Стоп. Не понял. Как так одни и те же результаты?? Наоборот, разные результаты показывает фотон, разве нет?
То, что частица ведет себя как волна , когда должна быть как частица, это другой результат, а не один и тот же.
Я ошибаюсь?

> Поп не имеет фактической экспериментальной базы в основе своих верований, в отличии от физики. Все сравнения с религией от тупости и невежества.
Большая проблема , для гуманитария непреодолимая. Физик предоставляет формулу, подкрепленную расчетами, поп показывает пальцем в небо и убедительно красноречив. Гуманитарий воспринимает мир сквозь эмоции, а не логику, матанализ для него похож на схождение благодатного огня.
> И вообще чтобы что-то понять, нужно хотя-бы начать обмазываться вменяемым научпопом.
Вот! Нет его. Научпоп пишут гуманитарии, а от этого он к науке имеет такое ж отношение, как я. Никакое.
Аноним 08/06/18 Птн 19:32:54  440436
>>440422
Легко! Берется и пишется теория всего.
А если серьезно, то да, проблема. Годных детских писателей очень мало, например.
Аноним 08/06/18 Птн 19:37:09  440437
>>440429
Не буду. Повторюсь, проблема с изложением/ восприятием виной.
Аноним 08/06/18 Птн 22:09:03  440443
>>440435
>То, что частица ведет себя как волна , когда должна быть как частица, это другой результат, а не один и тот же.
>Я ошибаюсь?
Уравнения описывают оба варианта развития событий, при этом, первый например похож на волны, второй на частицы. Но в самих уравнениях нет этого вот переключателя, там всё гладко и прекрасно, а подобные "приколы" возникают скорее из-за математики.
Аноним 08/06/18 Птн 23:30:54  440446
p01h9j58.jpg (35Кб, 600x450)
>>440435
>Стоп. Не понял.
https://www.youtube.com/watch?v=niOPGjuXdBg&t=2s

>Вот! Нет его. Научпоп пишут гуманитарии
Это потому, что обмазываешься собачьим говном, а не научпопом даже минимального уровня. Ты попробуй хотя-бы посмотреть видео с различными физиками-преподавателями, навскидку: Ахмедов, Казаков, Чирцов, Шестопалов, старичок Бояршинов и т.д. Хотя хуй знает что ты там поймешь после ВВС и прочего
Аноним 09/06/18 Суб 01:28:01  440448
Верно ли, что в любом кубометре атмосферы не считая газы из вулканов, недр итдна высоте до 1км найдётся атом, прошедший через нас (мы им дышали/ссали/срали)?
Аноним 09/06/18 Суб 08:32:54  440452
Нет ведь удачи с научной точки зрения.
Но как объяснить почему я спал проснулся повернулся на другую сторону кровати заснул, а на мою потолок упал, при том трещин не было
Аноним 09/06/18 Суб 08:58:58  440453
>>440443
а подобные "приколы" возникают скорее из-за математики.
В этом случае напрашивается вывод об несовершенстве математического анализа. Не, я не о том, что математика не нужна, я рассуждаю так: сама математика логична и способна описать большинство событий. Человек опирается на нее, познавая окружающий мир. Мы считаем единицами, но исходно это лишь метод и способ создания модели, помогающей понять окружающий мир, то есть это все возможно то, как микроб, сидя в заднице бегемота, в шестьнадцатиричной модели счисления осознает, где он находится, что есть тело, двигающееся в пространстве и вокруг много таких и не совсем таких тел, и все подчинено импульсам и векторам, в то время как бегемот, крутя хвостом, метит территорию. А сам микроб не может понять что он тупо в заднице.
Аноним 09/06/18 Суб 09:04:36  440455
>>440446
Видео не то. Брайан и Хокинг вызывают гораздо большее доверие. Хоть гуманитарию их читать тяжело. Однако, обмазываться другими, да еще и видео, не, нафиг нужно.
Аноним 09/06/18 Суб 09:06:30  440456
>>440453
Не совсем понял что ты хотел сказать. Тут скорее проблема восприятия и ограниченности человеческих мозгов, нежели математики.
Аноним 09/06/18 Суб 09:15:57  440457
>>440448
В тебе самом найдется атом, которым не только дышал и срал великий Карлсон , но и часть звезды, ярко горевшей каких то пару сотен миллиардов лет назад
Аноним 09/06/18 Суб 09:25:51  440458
>>440456
Именно это и хочу сказать, что возникновение и описание теории дуализма может быть таким от того, что мы считаем кучу арбузов, в качестве единицы принимая ну скажем, грабли. Со всеми зубчиками.
И потому проявление в нашем восприятиифотона то таким, то эдаким, следствие неспособности сознания воспринять его как что то единое, но выдающее противоречивый с нашей точки зрения результат. Не наркоманский трип, а несовершенность восприятия, ну и осмысления.
Аноним 09/06/18 Суб 09:37:15  440459
>>440458
Да блять. Не воспринимают его то таким то эдаким. Перестань. Его очень конкретно воспринимают.
Аноним 09/06/18 Суб 10:29:04  440461
>>440430
>линз
бамп
Аноним 09/06/18 Суб 10:32:17  440462
>>440430
Ну смотри какое дело, тут работает принцип "как входит так и выходит". Если ты понимаешь о чём я.
Аноним 09/06/18 Суб 10:47:34  440464
>>440455
>Видео не то.
>большее доверие.
Тогда записывай: Рыбников, Катющик, Ацюковский.
Аноним 09/06/18 Суб 10:59:27  440465
283581.jpg (54Кб, 725x508)
>>440457
>каких то пару сотен миллиардов лет назад
ты хоть понял, как обосрался
Аноним 09/06/18 Суб 11:33:13  440467
>>440459
Меня умиляет ваше снисхождение.

Каким его воспринимают?
Аноним 09/06/18 Суб 11:35:37  440468
>>440464

> Тогда записывай: Рыбников, Катющик, Ацюковский.
Ой ой. Гранмерси. Прибью их портреты на стену. Рядом с Мильтоном, Гомером и Паниковским.
Жирно.
Аноним 09/06/18 Суб 11:37:54  440469
>>440465
Разница между десятками и сотнями миллиардов так незначительна в контексте борьбы с мировым капитализмом, что ей можно пренебречь
Аноним 09/06/18 Суб 11:39:01  440470
>>440452
Можно объяснить тяготением например. Наличие трещин и их свойства в формулу можно не пихать.
Аноним 09/06/18 Суб 11:40:11  440471
>>440467
Как квант электромагнитного поля. Там квантование полей, ебануться можно пока разберешься
Аноним 09/06/18 Суб 11:42:10  440472
>>440471
Наконец то вменяемый ответ. Спасибо, пошел курить
Аноним 09/06/18 Суб 11:44:55  440473
>>440457
Так это понятно, а то нет. А ещё интересно, если на нашу планету падали инозвёздные тела, есть ли в нас от них что-то, много ли таких людей, чувствуют ли себя как-то по-особенному
Аноним 09/06/18 Суб 11:45:32  440474
>>440472
Когда говорят о "дуализме" имеют ввиду, что в некоторых случаях, уравнения волн очень близки к классике, где частицы это точки с массой.
Как например уравнение sin(x) вблизи нуля себя ведет как y=x;
Аноним 09/06/18 Суб 12:18:03  440475
>>440473
> Так это понятно, а то нет. А ещё интересно, если на нашу планету падали инозвёздные тела, есть ли в нас от них что-то, много ли таких людей, чувствуют ли себя как-то по-особенному
Если в башку влепиться метеор, то чувствовать себя будешь исключительно по особому.
А так даже говно внутри нас имеет романтичное происхождение
Аноним 09/06/18 Суб 12:26:31  440476
>>440474
В принципе, едва начав курение теории поля, сразу и становится очевидным ответ на мой вопрос о дуализме, о проявлении фотона то таким, то эдаким. Еще раз спасибо тому анону, который отослал меня на поле чудес
Вывод- анонтяи, сидящие здесь в ожидании фриков от науки, ни на что не годны. Обленились и зажирели.
Аноним 09/06/18 Суб 12:53:48  440478
1369673702490.jpg (132Кб, 768x1024)
>>439790 (OP)
Гипотетическая ситуация, пигмей переспал с европеоидом-карликом. Какова вероятность, что в результате на свет появится европеоид очень маленького роста и все его потомки тоже будут маленькими?
Как вариант, европеоид-карлик перспал с эскимосом. На самом деле я просто хочу вывести расу хоббитов с европеоидными чертами лица.
Карлик+пигмей=? Аноним 09/06/18 Суб 12:54:19  440479
1369673702490.jpg (132Кб, 768x1024)
>>439790 (OP)
Гипотетическая ситуация, пигмей переспал с европеоидом-карликом. Какова вероятность, что в результате на свет появится европеоид очень маленького роста и все его потомки тоже будут маленькими?
Как вариант, европеоид-карлик переспал с эскимосом. На самом деле я просто хочу вывести расу хоббитов с европеоидными чертами лица.
Аноним 09/06/18 Суб 15:18:47  440482
>>440479
Зависит от того, откуда у европеоида карликовость, если наследованная, то дети будут карланами. Адресок черкани, где карликов держишь. С ув., т.м.
Аноним 09/06/18 Суб 15:25:56  440483
>>440452
В этот раз не упал, в другой - на тебя упадёт и потолок, и все девять этажей сверху. Вот и вся удача.
Аноним 09/06/18 Суб 15:39:23  440484
>>440462
ты ошибаешься
Аноним 09/06/18 Суб 16:27:40  440488
15285501746820.jpg (24Кб, 480x362)
>>440482
Вопрос был не про карланность, а про евпропеиодные черты лица. Ок, т.е. по сути пигмеи не имеют решающей роли в данном эксперименте и достаточно найти карликов с определенной наследственностью.
Но у карликов, в отличии от пигмеев, пропорции очень специфические. Поэтому придется скрещивать пигмеев или эскимосов с обычными европеоидом. Какова вероятность в этом случае вывести расу европеоидных хоббитов? Записывайте - улица Пушкина, дом Колотушкина. Спросить Джона Рональда.
Аноним 09/06/18 Суб 16:46:27  440490
>>440488
50% Оставайтесь на месте, наряд выехал.
Аноним 09/06/18 Суб 16:46:29  440491
Тупой вопрос - неужели кто-то может на такое повестись? https://hi-tech-kushelev-group.nethouse.ru
Аноним 09/06/18 Суб 17:01:32  440492
image.png (250Кб, 667x824)
>>440491
Ты как будто вчера родился и не знаешь про cognitive bias и маркетинг.
Аноним 09/06/18 Суб 17:02:43  440493
>>440433
поле передается со скоростью света, но в проводе у тебя не поле а проводник со свободными
электронами внутри
а вот оптоволокно уже передает электромагнитное волны
загугли и найди отличие проводов от оптоволокна
Аноним 09/06/18 Суб 17:10:53  440494
4563452523.jpeg (10Кб, 276x183)
>>440490
>наряд выехал
Ок. Нам пора, дорогая.
Аноним 09/06/18 Суб 17:14:29  440495
>>440433
Можешь представить это как звуковые волны в плотной среде. Молекулы среды могут даже практически не двигаться, но при этом звуковая волна распространяется вполне быстро.
Аноним 09/06/18 Суб 17:28:19  440496
Есть ли хоть один научный факт, вроде фактов о вреде мяса, курения, алкоголя, жирной пищи, онанизма, лекарств, наркотиков, который можно было бы вот прям сейчас взять и проверить? Чтоб можно было бы найти научную серьезную работу, где были бы статистические данные, был бы расписан ход исследования, где проводились, как проверяли?
Аноним 09/06/18 Суб 17:31:43  440497
>>440496
https://www.ncbi.nlm.nih.gov/pubmed/
Наслаждайся.
Аноним 09/06/18 Суб 17:39:49  440498
>>440497
Спасибо конечно, но есть ли что-то из настоящей, непендосской науки стволовых клеток и прочего искусственного интеллекта?
Аноним 09/06/18 Суб 17:51:04  440499
>>440498
>настоящей
>непендосской
ты уж определись.
Либо это: https://www.ncbi.nlm.nih.gov/pubmed/
Либо это: http://www.aif.ru/health/pro

Английский - язык мировой науки. Специально для тебя никто не будет переводить все статьи французских, китайских, японских, немецких и всех остальных стран учёных, которые публикуют статьи на английском, чтобы не жить в своём маня-мирке, а идти в ногу со временем и иметь возможность принести пользу обществу и быть услышанным.
Аноним 09/06/18 Суб 22:01:10  440506
Почему при недосыпе хуй встаёт?
Аноним 09/06/18 Суб 22:41:12  440507
Вот допустим, у нас есть экзотермическая реакция, при которой 2 частицы (молекулы или атома) соединяются в одну.
Каким образом будет выделена получившаяся энергия?

По закону сохранения импульса она не может выделиться в виде скоростей осколков: сумма импульсов должна оставаться неизменной.

Электроны в каком-то атоме перейдут в более возбуждённое состояние? (а это вообще возможно, когда валентные электроны связаны?) И да, возбуждён будет случайные атом из из получившейся молекулы, или как?

Или атомы в получившейся молекуле будут с огромной скоростью притягиваться и отталкиваться? (считаем, что выделившейся энергии не хватает на разрыв связи)
Аноним 09/06/18 Суб 22:59:21  440509
>>440507
>>440507
>Каким образом будет выделена получившаяся энергия?
Полученная частица, определенно будет не очень устойчива, так как имеет энергию на развал обратно. Но это будет не продукт, а именно возбужденная молекула продукта.
В молекуле есть три типа уровней, электронные, колебательные и вращательные. Чаще всего конечно возбуждаются в таких реакциях вращательные, но бывает что и электронные уровни возбуждаются. Далее молекула просто встречает другую молекулу, и дает ей кинетической энергии.
>Электроны в каком-то атоме перейдут в более возбуждённое состояние?
В молекуле атомарные уровни уже не сохраняются, и возникают молекулярные орбитали и уровни.
>Или атомы в получившейся молекуле будут с огромной скоростью притягиваться и отталкиваться?
Ты схематично колебательный уровень описал. Но пока он не передаст энергию куда-нибудь, у него ТОЧНО хватит её чтобы развалиться. Ну или банально нарушение закона сохранения.
Аноним 09/06/18 Суб 23:11:19  440510
>>440457
>но и часть звезды, ярко горевшей
Собственно, единственное место во вселенной, где могут возникнуть атомы тяжелее железа - это сверхновые. То есть весь йод в щитовидке, например - гарантированно из звёзд.
Аноним 10/06/18 Вск 01:33:41  440521
Существует ли электролиз с поглощением тепла?
Ну тоесть любой электролиз эндотермическая реакция, но есть ли такой который поглощает не только электрическую энергию но еще и тепло?

Существует ли гальванический элемент, при работе которого будет частично идти эндотермическая реакция?
Аноним 10/06/18 Вск 02:23:52  440524
Расскажите про коллапс волновой функции на пальцах, знаю только школьный курс физики.
Аноним 10/06/18 Вск 13:11:54  440534
>>440492
Кек, типичная новость
>ДИЗАЙНЕР РАЗРАБОТАЛ новая вещьнэйм
Аноним 10/06/18 Вск 13:24:32  440538
>>440534
Потому что design в инглише один - инженерный ли, архитектурный ли, визуальный ли. А дебилы, которые делают типичные переводы для типичных вау-новостей, переводят под калечку не включая мозга. Important? Значит, что-то про импорт. В таком ключе.
Аноним 10/06/18 Вск 13:34:13  440539
>>440506
Бамп вопросу.
Аноним 10/06/18 Вск 15:48:36  440550
>>440474
Когда говорят о дуализме этого в виде не имеют.
Там вполне себе дуализм. В классической волновой теории - там волны. Без дуализма. Несмотря на приближения...
Аноним 10/06/18 Вск 16:01:46  440551
>>440521
>любой электролиз эндотермическая реакция
Не совсем.

>Существует ли гальванический элемент, при работе которого будет частично идти эндотермическая реакция?
Термодинамика не запрещает.
Аноним 10/06/18 Вск 16:30:56  440555
Когда я свободно опускаю кувалду на камень, камень легче расколоть, чем когда я добавляю к кувалде усилие. И чем больше усилие, тем неподатливее камень. Почему так?
Аноним 10/06/18 Вск 16:39:49  440556
>>440555
руки не из того места растут
Аноним 10/06/18 Вск 16:44:52  440557
>>440556
Тупые вопросы разрешены, но не тупые ответы!
Аноним 10/06/18 Вск 16:51:46  440558
>>440550
Когда говорят о дуализме, я боюсь мы вообще понять не можем что имеют ввиду.
>В классической волновой теории - там волны. Без дуализма. Несмотря на приближения...
Уравнения КМ волновые, в то же время переходят в классику при h->0
Аноним 10/06/18 Вск 17:30:50  440559
>>440557
когда кувалда падает свободно, она падает под свои весом с ускорением g. Если ты приложишь дополнительную силу вниз, то скорость падения будет увеличиваться. Но из-за рукожопости может казаться, что напряжение мышц ускоряет предмет, а на самом деле наоборот. В итоге устаешь больше, а результат меньше.
Аноним 10/06/18 Вск 17:47:53  440560
>>440409
ну ответьте уже что нибудь пожалуйста.
Аноним 10/06/18 Вск 18:04:43  440561
>>440560
Тоже в пабмеде можешь поискать, но тема очень уж пикантная, поэтому большинство адекватов за неё не возьмуться - не дай боже найдут какие-то рассовые отличия, их потом с говном смешают и крест на карьере поставят. Так что из тех немногих статей, что есть, многие написаны неадекватами, которые хуй клали на всю научную методологию. Где-то вероятно есть и нормального уровня статьи, но я хуй знает. Короче, это большая редкость.
Аноним 10/06/18 Вск 18:18:04  440562
Объясните, почему гипноз не существует. Или существует. Желательно без игры в слова.
Аноним 10/06/18 Вск 18:21:50  440564
>>440562
Потому что язык - это охуенный инструмент, а инструментом можно так ёбнуть по голове, что мало не покажется. Без игры в слова на википедию пиздуй.
Аноним 10/06/18 Вск 18:30:21  440567
>>440564
Я ж не зря спрашиваю на доске про НОУКУ. С позитивистских позиций, с двойными слепыми тестами и прочим говном - существует? Нет?
Аноним 10/06/18 Вск 18:30:38  440568
>>440561
Спасибо, анонче
Аноним 10/06/18 Вск 18:35:07  440569
>>440567
Зайди на гугл академию, и погугли там. Я лично хз.
Аноним 10/06/18 Вск 19:44:08  440575
>>440567
>С позитивистских позиций, с двойными слепыми тестами и прочим говном - существует? Нет?
Друг, гипноз используют в операциях вместо наркоза. Пациент в сознании, его режут и он ничего не чувствует. На википедии есть соответствующая статья, со ссылками и пруфами. "Гипнохирургия" или "гипнонаркоз", как-то так. Каких хуёвслепых тестов тебе надо?

То, что существует сценический гипноз, в котором имеет место представление и клоунада - ну так и анатомические театры тоже существуют, можно пойти поржать. И что, анатомия ненаучна теперь от этого стала?
Аноним 10/06/18 Вск 20:14:45  440582
>>440575
>Гипнохирургия
>На википедии
>It is claimed that hypnosis for anaesthesia has been used since the 1840s where it was pioneered by the surgeon James Braid.[citation needed] There are occasional media reports of surgery being conducted under hypnosis,[1][2] but since these are not carried out under controlled conditions, nothing can be concluded from them.
Ну и?
Аноним 10/06/18 Вск 20:49:08  440585
>>440559
Нет, ты не прав. То что предмет ускоряется, это факт, подтверждаемый секундомером, то есть это не кажимость. А результат меньше, вот в чем штука!
Аноним 10/06/18 Вск 23:43:06  440601
>>440585
>То что предмет ускоряется, это факт, подтверждаемый секундомером, то есть это не кажимость.
Значит в снятых показаниях секундомера погрешность, попробуй автоматический секундомер, который срабатывает на пересечение лазера.

Напомните, какое отношение генов к воспитанию согласно последним исследованиям в причинах самореализации?
Аноним 11/06/18 Пнд 00:14:11  440602
>>440601
>какое отношение генов к воспитанию
30 на 60
Аноним 11/06/18 Пнд 02:33:26  440605
>>440602
Всмысле 1 к 2 или ты ещё 10 забыл?
Аноним 11/06/18 Пнд 04:39:09  440607
Искуственная нейронная сеть — нелинейный классификатор, потому что функция активации нейрона нелинейна, но дифференцируема, из этого и происходит вся магия. Это понятно. Но из этого ни разу не следует, что получившаяся система может аппроксимировать самые разные нелинейные функции и хотя по результатам применения NN понятно, что это, видимо, так, мне нихуя не очевидно, что NN может аппроксимировать и сколько слоёв/нейронов для этого нужно.

То есть, короче: очевидно, что однослойный перцептрон Розенблатта это тупо линейная регрессия, но при добавлении слоёв система вроде как становится способна аппроксимировать разные нелинейные функции. Может ли (в принципе) многослойный перцептрон аппроксимировать любую функцию и для каких функций достаточно скольки слоёв?

Есть какое-нибудь краткое и понятное объяснение вот этого вот конкретно момента? Исключая вопросы практичности и обучения, рассматривая только саму вычислительную способность нейронных сетей.
Аноним 11/06/18 Пнд 08:30:41  440608
>>440607
Теоретически может любую, вопрос лишь в том, сколько для этого понадобится нейронов (теоретически даже однослойный может любую, но на практике ему для сколь-нибудь сложных систем может потребоваться столько нейронов, сколько на земле не существует, так что многослойные рулят).
Сколько слоёв - никто не знает. Чувак, никто толком вообще не понимает почему эта хуйня работает и каким именно образом вот эта конкретная сетка научилась решать вот эту конкретную задачу. Т.е. теории-то есть, но понимания нет.
Потому что как правило речь идёт о сотнях тысяч параметров, разобраться в которых мясной NN в нашей черепушке непосильно. А тем более для каждого обучения на тех же самых параметрах в разной последовательности получаются разные результаты, которые могут быть очень близки по точности.
Тут даже до ReLU функции активации додумались сравнительно недавно, а ведь она гораздо проще традиционной сигмоиды. Но при этом она ускоряет обучение и уменьшает потребности к количеству нейронов на порядки.
Аноним 11/06/18 Пнд 08:47:09  440609
>>440608
Это всё пустая болтовня, а ты — пустой болтун, ничего не понимающий в теме. Вопрос чисто математический: что можно аппроксимировать и сколько слоёв нужно это строгий, доказуемый факт, а не "никто не знает", блядь.

Впрочем, хотя я не нашёл единого внятного объяснения, пока я гуглил, выяснилось что всё печально и разбирать тут особо нечего, потому как всё упирается в UAT, и по сути аппроксимация получается такая же, как с полиномами, и с конечным числом параметров на самом деле аппроксимируется приблизительно нихуя. То есть, нет, я не могу научить нейронку вычислять не то что произвольную, но даже самые привычные со школьной скамьи функции.

Другое дело, что это заставило меня задуматься, что ведь другой аппроксимации в природе похоже вообще просто нет. Это мне кажется, что я могу производить аналитические вычисления с нелинейными функциями, но это просто потому что нейронов у меня в голове дохуя, а на самом-то деле в конечном счёте всё точно так же сводится к локальной аппроксимации. Вот это неожиданный, конечно, вывод, надо будет осмыслить.
Аноним 11/06/18 Пнд 09:20:44  440610
>>440609
Математически любую непрерывную функцию можно аппроксимировать одним скрытым слоем: https://en.wikipedia.org/wiki/Universal_approximation_theorem
Аноним 11/06/18 Пнд 10:05:01  440611
>>440610
> можно аппроксимировать
Локально. И
> всё упирается в UAT
Аноним 11/06/18 Пнд 11:14:43  440612
>>440506
Бамп вопросу.
Аноним 11/06/18 Пнд 11:44:36  440615
>>439790 (OP)
Есть уравнение непрерывности, дивергенция плотности тока равна нулю, получается, что плотность тока постоянный вектор? Но если сечение проводника меняется, то плотность то же меняется же?
Аноним 11/06/18 Пнд 11:49:17  440617
>>440615
В дивергенции есть еще две компоненты, поэтому когда у тебя проводник сужается. то появляется проекции плотности тока по осям y и z, при этом на тонком участке плотность тока опять же постоянна.
Аноним 11/06/18 Пнд 11:53:18  440618
122.png (6Кб, 308x176)
>>440617
Вот тут например в сечениях 1 и 2 нету же других компонент
Аноним 11/06/18 Пнд 12:04:09  440619
>>440618
В сечениях дивергенция плотности тока 0. Почему ты считаешь, что если дивергенция 0, то обязательно вектор постоянный?
Аноним 11/06/18 Пнд 12:22:27  440620
>>440619
В сечениях плотность имеет только одну компонентк, например Jx1 и Jx2, divJ=dJx/dx=0=>Jx=const?
Аноним 11/06/18 Пнд 12:29:32  440621
1. Почему изображние насилия/смерти более допустимо, чем изображение секса/наготы? Почему так сложилось? Почему это хорошо и правильно?
Хард мод: обойдитесь без объяснений "тупые веруны" и "дети видят на экране сиськи и тут же принимаются ебаться".
2. Бамп: >>440562
Аноним 11/06/18 Пнд 12:38:52  440622
1
Аноним 11/06/18 Пнд 12:47:17  440623
>>440620
Да, только одну. Теперь двигай медленно сечение 2 влево, до сечения 1. И смотри что происходит в плотностью тока.
Аноним 11/06/18 Пнд 12:49:49  440624
>>440623
>что происходит в плотностью тока
В сужении будет изменяться. А если не двигать, ток через сечение 1 и 2 должны быть одинаковыми, значит плотность тока должна быть разной, но почему тогда дивергенция равна нулю?
Аноним 11/06/18 Пнд 12:52:27  440625
>>440624
>>440624
>ток через сечение 1 и 2 должны быть одинаковыми
Ты это выводишь из предположения, что у тебя во всём проводнике одна компонента.
Но в месте, где изменения происходят, у тебя больше компонент.
У тебя и в сужении дивергенция 0, если что.
Аноним 11/06/18 Пнд 12:53:44  440626
>>440625
Че ток разный в сечениях что ли?
Аноним 11/06/18 Пнд 12:55:38  440627
>>440626
Ток один. плотность разная.
Аноним 11/06/18 Пнд 12:56:58  440628
>>440627
>плотность разная
Почему дивергенция тогда нулевая то?
Аноним 11/06/18 Пнд 12:57:22  440629
>>440627
Или нельзя забивать на область сужения?
Аноним 11/06/18 Пнд 13:00:04  440630
>>440628
>>440629
У тебя эти сечения разделены сужением. Конечно нельзя забивать. Ты же все рассуждения построил на том, что у тебя на участке одна компонента.
>В сечениях плотность имеет только одну компоненту
Представь автомобиль, который едет с постоянной скоростью прямо, потом поворачивает с такой же скоростью, и дальше по прямой едет. Везде кроме поворота производная по оси будет постоянна. Но ты же не можешь закрыть глаза на поворот.
Аноним 11/06/18 Пнд 13:04:09  440631
>>440630
Понял. Спасибо.
Аноним 11/06/18 Пнд 14:05:35  440632
Street Fighter [...].webm (1044Кб, 1920x1080, 00:00:04)
Может ли на данный момент существовать нейросеть, что в состоянии действовать в видеоигре, а в частности в данном файтане, на уровне хорошего игрока?

я понятия не имею, что такое нейросеть
знаю, что можно разобраться за 30 сек. банально лень усваивать новую информацию
Аноним 11/06/18 Пнд 14:30:17  440633
>>440632
>что в состоянии действовать в видеоигре, а в частности в данном файтане, на уровне хорошего игрока?
Ты и об игровых ИИ понятия не имеешь.
ИИ делают для того, чтобы болванчики тупили реалистичнее, а не для того, чтоб они без ошибок хуярили.

Берёшь третьекваку, включаешь там самого сильного бота и принимаешь хуи в сраку. С шахматами то же самое. И никакого ИИ.
Аноним 11/06/18 Пнд 14:45:50  440634
>>440633
Так тут дело вот в чем. Я банально хочу поглазеть на бои ботов в этой игре, как в старых файтанах, но понятия не имею как подключить игровой "ИИ" (или вообще любой другой ИИ) со своей стороны. Думал, может кто подскажет направление в какую сторону копать, дабы воплотить мое желание.
Аноним 11/06/18 Пнд 14:47:15  440635
>>440633
Ты для цивилизации запили охуенного бота без предоставления ему гандикапа, а потом кукарекай
Аноним 11/06/18 Пнд 15:51:27  440638
>>440621
>Почему изображние насилия/смерти более допустимо, чем изображение секса/наготы?
Потому что это не так.
https://en.wikipedia.org/wiki/Motion_Picture_Association_of_America_film_rating_system
>>440634
>понятия не имею как подключить игровой "ИИ" (или вообще любой другой ИИ) со своей стороны.
>знаю, что можно разобраться за 30 сек. банально лень усваивать новую информацию
https://www.youtube.com/watch?v=eWSbIXSbMis
Свободен.
Аноним 11/06/18 Пнд 17:58:39  440640
Господа, помогите пожалуйста. Не знаю, в правильном ли я месте задаю вопрос, не нашел тред для матемитиков.

В экселе когда функция экстраполируется (мнк) выводится коэфициент соответствия (коэф. корреляции Пирсона, он же Эр Квадрат). Подскажите, по какой формуле считается этот коэффициент при условии, что функция не имеет свободного члена . То есть как я понимаю, когда считается мнк сумма квадратов разности дифференцируется только по к, у нее R^2 отличается от того, который расчитывается для функции с свободным членом (которая дифференцируется по частным производным).

Короче где достать эту модифицированную формулу расчета? Выручайте, господа
Аноним 11/06/18 Пнд 18:02:20  440641
>>440638
>https://en.wikipedia.org/wiki/Motion_Picture_Association_of_America_film_rating_system
Я смотрел известную документалку про эти рейтинги, цензоров волнует там прежде всего секс и мат. Каких-то критериев нет, всё решает комиссия.
В эфирном ТВ (что в США, что в РФ) нет никаких сисек, а смерти и насилия сколько угодно, в США вообще устраивают шоу из реальных массовых убийств, мэнхантов и т.д.
Аноним 11/06/18 Пнд 18:02:51  440642
>>440635
Цивилизация это тупо шахматы + весёлая ферма. И для того, и для другого боты есть, вопрос только в том, что это никому нахуй не надо.
Аноним 11/06/18 Пнд 19:36:06  440643
>>440638
>Свободен.

Обоссан. Нихуя не пояснил и за каким-то хуем приплёл первое, что выдал гугол.
Аноним 12/06/18 Втр 02:12:27  440656
>>439790 (OP)
Кто украл носки у Филателиста?
Аноним 12/06/18 Втр 02:38:51  440665
>>440656
Хохлы
Аноним 12/06/18 Втр 17:07:24  440702
>>440384
Бамп вопросу.
Аноним 13/06/18 Срд 00:29:04  440719
>>440702
Если биполярный транзистор в процессе работы устройства нагревается, то ток коллектор-эмиттер увеличивается, то есть температурный коэффициент сопротивления у биполярных транзисторов отрицательный.

У полевых же все наоборот — температурный коэффициент сток-исток положительный, то есть с ростом температуры растет и сопротивление канала, то есть ток сток-исток уменьшается. Это обстоятельство дает полевым транзистором еще одно преимущество перед биполярными: полевые транзисторы можно без опаски соединять параллельно, и не потребуются выравнивающие резисторы в цепах их стоков, поскольку в соответствии с ростом нагрузки станет автоматически расти и сопротивление каналов...


Я понятия не имею как попал на эту доску, но решил нагуглить это для тебя. Это ответ?
Аноним 13/06/18 Срд 06:51:00  440731
>>440638
Пиздец ты даун.
Аноним 13/06/18 Срд 09:33:37  440737
>>440621
по первому вопросу.
второе из первого пункта.
государство это аппарат для стрижки скота его контроля. если не обученным пользоваться гандонами пиздюкам круглосуточно крутить порево, они, ВНЕЗАПНО, станут повторять за взрослыми. что приведет к неконтролируему пиздецу в плане рождаемости и вообще может не слабо так наебнуть их здоровье по молодости.
а этим пиздюками надо еще становиться винтиками машины, учиться, служить, платить налоги и сдохнуть в нищете или где-нибудь на сцене театра военных действий на потеху верхушке власти.
по второму - я на счет гипноза не знаю, но словами до самоубийства довести как нехуй делать.
Аноним 13/06/18 Срд 10:00:59  440740
>>440737
>порево
С чего ты взял, что сиськи это "порево"? Под запретом даже изображение не-сексуализированного обнаженного женского тела, а вовсе не инструкций по спариванию. Причем это не какая-то инерция и традиция, Фэйсбук запрещает постить женские соски. Фото общим планом пляжа уродливых и старых нудистов приравнивается к пропаганде ИГИЛ- почему?
Аноним 13/06/18 Срд 10:02:09  440741
>>440740
А ты напиши им в поддержку, объяснят.
Аноним 13/06/18 Срд 11:28:21  440743
>>440719
Спасибо что откликнулся, анончик. Да, ты все верно описал, но это скорей режим работы в высоких температурах. У полевиков же при превышении 200 градусов наступает тепловой пробой, то есть разрушение внутренней структуры. Вот и хотелось узнать почему же так - ведь там нет таких тонких пленок(МОП не в счет) как база в биполярном.
Аноним 13/06/18 Срд 11:38:49  440745
>>440740
потому что такие фото наводят не на те размышления, которые выгодны власти.
это вопрос из той же оперы - почему алкоголь продается, а трава - нет.
а ответ прост алкоголь угнетает нервную систему, а трава - нет.
а рабам позволено только быть угнетенными.
Аноним 13/06/18 Срд 11:47:53  440746
>>440740
>С чего ты взял, что сиськи это "порево"?
Любой триггер сексуального возбуждения аналогичен пореву - не важно хентай это или учебник по биологии.

Тут целый ряд причин:
- традиция, авраамические, да и индуистские религии к открытому сексу относятся плохо. Тут же инертность страха перед эпидемиями - в средние века открытые отношения были число по здоровью противопоказаны, но сейчас эта проблема решена.
- капиталистические, семья это не просто сожительство а экономический конструкт, который выгоден властям. Соответственно они будут форсить семейные ценности которые несовместимы с открытым сексом, даже в этих ваших Гейропах ЛГБТ во все дыры, но о пролигамии/полиандрии предпочитают не вспоминать. Наоборот, даже гей-отношения стараются перевести в брак.
- потреблядские, неудовлетворенность в любви или сексе стимулирует спрос на другие товары. Грубо говоря, если бы тети с детства видели сиськи и письки, как бы ты им толкнул очередной фильм или мультик с сексуальными позами? Это было бы просто неинтересно.
- наркоманские, любовь - наркотик, а наркотики запрещены.
- контроль рождаемости, по сути сексуальные табу это обратная связь для сексуального влечения. После всяких войн растет фрилав, при перенаселении и стабильности - скрепы.
- проблема собственности: для сексуальных отношений нужно иметь партнера в своей собственности(не полностью конечно, но отсылка к владению все же есть) - то есть в браке. До совершеннолетия дети - собственность их родителей, поэтому им секс тем более запрещен.
- социальная активность, у тех же ИГИЛ максимальные табу на секс для того чтобы средний шахид был агрессивен и шел воевать а не сливал тестостерон в пизду.
- банальное непонимание молодых старшим поколением, у 50-летнего депутата уже не стоит и он считает неправильным оргии и фетиши.

Аноним 13/06/18 Срд 11:54:04  440748
>>439790 (OP)
Если идти по аналогии с водой, любой проводник аналогичен пористому материалу(скажем, губке), сквозь который течет вода - отсюда и сопротивление. Соответственно аналог водопроводной трубы - это некий вакуумный канал для электронов. Получается такой канал был бы идеальный сверхпроводником без сопротивления, так в чем же подводные, анон? Почему вместо низкотемпературных сверхпроводников тупо не использовать пучки вакуумных наномикро-трубок? Какие подводные?
Аноним 13/06/18 Срд 12:08:46  440749
>>440746
>а не сливал тестостерон в пизду.
это вся суть обезьяньей природы.
вместо того, чтобы сливать таким образом, они друг друга надрачивают под разными предлогами на врагов внешних и внутренних.
чтобы в том числе продавать друг другу оружие для геноцида.
а могли бы как бонобо решать проблемы не дубиной по голове.
но я сомневаюсь, что такое общество вообще возможно одновременно с капитализмом, главная цель которого - извлечение прибыли, путем насаждения дефицита или его иллюзии.
Аноним 13/06/18 Срд 12:21:26  440750
>>440746
>Любой триггер сексуального возбуждения аналогичен пореву
Это ты уже имлицитно утверждал. Бездоказательно.
>традиция
Не аргумент. Традиция не рациональна и отражает отношения в традиционном (аграрном) обществе. Мы же живем с совсем другом.
В индуизме ебля прямо в оформлении храмов, и ничего.
>капиталистические, бла-бла-бла
Хорошо, примем, что что дети, посмотрев на еблю, будут безудержно ебаться. Это серьезное допущение, но для тебя оно не первое, оно держится на "любая нагота = сексуализированная нагота" и "любая нагота = инструкция по ебле и порнография". Это явная чепуха.
Аноним 13/06/18 Срд 12:30:31  440752
>>440746
>в средние века открытые отношения были число по здоровью противопоказаны, но сейчас эта проблема решена.
>но сейчас эта проблема решена.
Ты что, дебил?
Дальше не читал.
Аноним 13/06/18 Срд 12:30:41  440753
>>440750
>Мы же живем с совсем другом.
манямирок из 10.
Аноним 13/06/18 Срд 12:32:39  440754
>>440749
>но я сомневаюсь, что такое общество вообще возможно одновременно с капитализмом,
Что, коммунизм нужон?
Аноним 13/06/18 Срд 12:34:54  440755
>>440748
Ну хорошо, электроны. А что в трубке будет плюсом?
Аноним 13/06/18 Срд 12:54:09  440756
>>440753
>95% населения живет в деревне, крестьянским трудом, в расширенной семье, женщины рожают по десять детей, доступа к дешевым пром. товарам нет, антибиотиков и контрацептивов нет, поголовной грамотности нет, СМИ нет
Лалка, такого даже в самых отсталых углах Африки нет, мир радикально изменился.
Аноним 13/06/18 Срд 13:46:10  440757
Что будет, если в железный шар налить воду и скинуть в марианскую впадину?
Аноним 13/06/18 Срд 13:51:36  440759
>>440757
Ничего. Давление воды изнутри шара всегда будет равно давлению воды снаружи. Возможно, шар промнётся для компенсации.
Аноним 13/06/18 Срд 13:53:46  440760
Поясните за бихевиоризм, как наблюдатель может влиять,
демон Лапласа же просто поправка на нехватку данных по моему.
Почему отрицают что они это не их среда воспитание.
Количество опыта во многих сферах.
Ведь знать много нужной информации, получить полезный опыт, это единственный способ прекратить пускать пузыри жизни.
Мне просто так стало легче жить что где я не мог что-то сделать, то я не мог это сделать.
Я стал более свободен от детерминизма, ведь не надо больше плакать, жалеть себя.
Аноним 13/06/18 Срд 14:01:10  440761
>>440759
А если в атмосферу Венеры?
Аноним 13/06/18 Срд 14:08:36  440762
>>440748
Абсолютного вакуума не бывает. И пытаться поддерживать очень глубокий вакуум даже сложнее, чем околонулевую температуру.
И, кстати, с пористым материалом уместнее сравнивать полупроводники, а проводники как раз с трубами - в трубах тоже сопротивление есть и ламинарное разделение потоков (в центре течение сильнее, чем по краям).
Аноним 13/06/18 Срд 14:31:39  440763
>>440760
Я понял, я животное способное мыслить, нет ответственности за что я не так всё делаю как надо обществу, чтобы по Ницше быть человеком ведь ненужно.
Аноним 13/06/18 Срд 15:54:41  440768
>>440750
>Это ты уже имлицитно утверждал.
Ну докажи это сам. Блокируют контент по возможности сексуально возбудиться от его просмотра а не от названия.
>Традиция не рациональна
Я и не утверждал что рациональна. Но она есть.
>традиционном (аграрном)
Традиционное = аграрное?
>Мы же живем с совсем другом.
В нашем обществе нет религии, преобладающих моральных ценностей и прочих культурных конструктов?
>В индуизме ебля прямо в оформлении храмов, и ничего.
Да, но и там есть табу на это. Впрочем, не такие как у нас.
>Хорошо, примем, что что дети, посмотрев на еблю, будут безудержно ебаться.
Не обязательно но могут. Такая логика у запрещающих, это не мой личный вывод.
>"любая нагота = сексуализированная нагота"
Нет, тут утверждение "любая нагота = потенциально возбуждающая нагота"

>>440752
А что не так? Кондомы и противовирусные изобрели, ебаться без заражения возможно. По крайней мере для адекватов, в средние века даже у бережливых выбора не было.

>>440754
>Что, коммунизм нужон?
Не обязательно он , но что-то иное.
Аноним 13/06/18 Срд 16:08:16  440769
>>440755
>А что в трубке будет плюсом?
Так же как и в проводе - другой конец трубки.

>>440762
>пытаться поддерживать очень глубокий вакуум даже сложнее, чем околонулевую температуру.
Но чем меньше диаметр нашей трубки тем проще поддерживать такой вакуум. В конце-концов между атомами/молекулами тоже вакуум, и его никто не поддерживает.

>с пористым материалом уместнее сравнивать полупроводники, а проводники как раз с трубами
Нихуя, я тоже так думал. Проводники это те же пористые материалы только с гораздо большими ячейками. Если полупроводник это губка поролоновая, то проводник - это металлическая губка, которая уже ближе к решетке чем к губке.
Короче основная идея в том что любое вещество для электронов аналогично губке, и чем больше расстояние между атомами в кристаллической решетке тем меньше сопротивление. Конечно там еще и другие факторы влияют вроде заполненности верхних уровней орбитали, но не суть.
Соответственно если создать решетку с большим межатомным расстоянием, то будем ближе к трубке чем к губке, и в определенный момент сопротивление окажется нулевым т.к. тепловые колебания атомов не будут мешать электронам.

> в трубах тоже сопротивление есть
В середине трубы сопротивление равно нулю, оно только у стенок. А разделение потоков из-за вязкости воды, у электронов же этой проблемы нет - поток в центре трубки независим от краев(до определенной их концентрации конечно).
Аноним 13/06/18 Срд 16:12:51  440771
>>440769
Ты выдумал не самую подходящую абстракцию и начинаешь её обсасывать. При этом абсолютно не воспринимаешь то, что тебе говорят.
Так что вперёд, гений, пили свой электронопровод. Не вижу смысла убеждать человека, который хочет только высказаться, а не выслушать.
Аноним 13/06/18 Срд 16:42:13  440772
>>440768
Кондомы не гарантируют безопасность. И, кстати, приятных тебе мандавошек и переносимых ими приятных вещей. Лалка.
Только сложное половое поведение гарантирует защиту от зпппппп.
Аноним 13/06/18 Срд 17:25:46  440773
>>440771
>Ты выдумал не самую подходящую абстракцию
Подскажи тогда другую. с этой абстракцией хоть понятно как работает транзистор(и биполярный и полевой), и вообще почти вся электроника.

>Не вижу смысла убеждать человека, который хочет только высказаться, а не выслушать.
Ну и нахуй ты слился? Норм же обсуждали.
Аноним 13/06/18 Срд 17:28:23  440774
>>440773
>Подскажи тогда другую.
Проводники и электроны.
>Норм же обсуждали
Ну так ты и не обсуждал, ты софизмом занимался.
Абстракции подходят только для того, чтобы получить упрощённое понимание. А если на этом упрощённом понимании пытаться делать выводы - это уже подмена понятий и софистика.
Аноним 13/06/18 Срд 18:05:11  440775
>>440768
>Традиционное = аграрное?
Да, все традиционные общества аграрные. Современный мир переживает отмирание индустриального общества. К примеру, все говорят о смерти нуклеарной семьи - но ведь нуклеарная семья это Модерн, это продукт урбанизации, ничего в ней нет ничего "традиционного", её 200-300 лет назад просто нигде не было.
>В нашем обществе нет религии
В нашем, российском - организованной религии практически нет. Церковь ежемесячно посещает что-то около 4% населения (по преимуществу пожилого).
>преобладающих моральных ценностей
"Моральные ценности" есть везде, как и "культурные конструкты". Какая тут связь с чем-либо?
>там есть табу
Ебутся прямо на фасадах, какие табу?
>любая нагота = потенциально возбуждающая нагота
Как и любое изображение голых ног или там поцелуев. Опять взятые с потолка утверждения.
Аноним 13/06/18 Срд 18:29:34  440776
>>440774
>Проводники и электроны.
Электрон нельзя пощупать и увидеть.

>А если на этом упрощённом понимании пытаться делать выводы - это уже подмена понятий и софистика.
Так я и не напрямую выводы делал, в вакууме действительно электронам проще перемещаться, а межатомное расстояние влияет на сопротивление.
Аноним 13/06/18 Срд 18:37:25  440778
>>440776
Проще-то может и проще, но как я уже сказал - обеспечение герметичности потребует много затрат, а помимо неё тебе нужно удерживать электроны в узком канале. Электроны друг друга не любят и стремятся разлететься, так что тебе нужно магнитное поле правильной конфигурации, чтобы удержать их или тем более чтобы их куда-то применить. И для создания магнитного поля тебе опять же нужен ток. Так что твой электронопровод из как тебе кажется всего-лишь герметично запаянной трубки превратиться в комплекс, чуть поменьше LHC.
Аноним 14/06/18 Чтв 16:27:43  440804
image.png (204Кб, 1280x1024)
>>439790 (OP)
Математики! Хуле сложные проценты такие сложные?!! Как мне посчитать набегающую сумму, если раз в год к ней будет добавляться один и тот же процент, но не от стартовой суммы, а от текущей после начисления процентной суммы за текущий год?
Аноним 14/06/18 Чтв 17:47:14  440806
>>440804
f[x] = f[x-1] + p f[x]
f[x](1-p)=f[x-1]
f[x]=f[x-1]/1-p
f[1]=f[0]/(1-p)
f[x]=f[0]/(1-p)^x

f[x] - сумма
f[0] - стартовая сумма
x - год
p - процент
Аноним 14/06/18 Чтв 18:11:23  440809
Снимок.PNG (42Кб, 1295x144)
Распишите мне как он нашел вариацию, умаляю. Почему делить на n^2?
Аноним 14/06/18 Чтв 19:13:54  440813
Что вы думаете о Жаке Фреско?
Почему он говорил что образование должно быть универсальным? Ведь на дваче говорят что это невозможно.
Такое образование будет неэффективное ведь, или нет?
Аноним 14/06/18 Чтв 19:45:09  440814
>>440813
левак, утопист, сектант
define универсальным; ссылку на пост где говорят
define неэффективное; применительно к чему оно должно быть эффективным?
Аноним 14/06/18 Чтв 20:38:23  440815
>>440814
вот момент видео
https://youtu.be/V4vmgi-sRZQ?t=347 про универсалов.
Но понравилось это видео https://youtu.be/eipUFzkuhQQ?t=287
Аноним 14/06/18 Чтв 21:14:11  440817
>>440809
Давай с определения смотреть.
Мы ищем дисперсию средней величины. Поэтому тебе нужно будет взять M[(сумм(y)/n)^2]-(M[cумм(y)/n])^2 а дальше пораскрывать всё, и воспользоваться линейностью мат. ожидания.
Он как я понимаю обошел немного другими сторонами.
Я понимаю, что скорее всего тебе сложно
Аноним 14/06/18 Чтв 21:29:31  440818
>>440809
Я даже нашел тебе доказательство.
Аноним 14/06/18 Чтв 21:29:50  440819
>>440818
>>440809
https://ru.coursera.org/learn/probability-theory-basics/lecture/iXcW9/dispiersiia-summy-niezavisimykh-sluchainykh-vielichin
Аноним 14/06/18 Чтв 22:59:19  440823
Куда уходят волны, например радиоволны поглощаются средой, атмосферой и землей ведь так?
А куда уходят свч волны в микроволновке, ведь там металлическая поверхность и они отражаются от стенок микроволновки. Тоесть вся энергия волн поглащается самой пищей? Каким вообще образом происходит поглощение волн материей?
Аноним 14/06/18 Чтв 23:14:06  440824
>>439790 (OP)
https://www.popmech.ru/science/221401-kanadskiy-termoyad-na-parovoy-tyage-budushchee/
https://en.wikipedia.org/wiki/General_Fusion
Как рассчитать сколько энергии нужно чтобы склеить охапку водорода в гелий по такому методу?

Я имею ввиду, можно же тупо наделать стоячих волн частота которых соотносится друг с другом как простые числа, которые раз в сто лет будут сходится и нагревать@сближать материал в центре мишени. Можно даже будет тупо нагнать водорода под давлением и напрямую его в гелий перегонять, без регистрации и жидких свинцовых сфер.
Аноним 15/06/18 Птн 00:07:39  440829
>>440819
Эта курсера проклятая блокирует видос после пробных 5 минут просмотра, а после регистрации не дает смотреть видосы, которых нет в первой неделе курса. Щас я распишу как я это понимаю, чтобы все здесь проиграли. Как ты уже написал:
σ^2 = E[Xi^2] - μ^2, это уже следствие линейности мат. ожидания.
Далее, нам известно что это выборка по средним величинам значит каждое Xi = μ и E[μi^2] = (n μi^2)/n = μ^2
σ^2 = μ^2 - μ^2 =0, и никакой дисперсии вообще нет, потому что каждая из сучайных величин равна мат. ожиданию выборки. Теперь я хочу услышать о какой дисперсии по выборке средних вообще идет речь и почему я ее не правильно понимаю.
Аноним 15/06/18 Птн 01:26:08  440831
>>440815
Кажется под универсалами он имеет в виду историков, лол. Но если смотреть общий контекст, то его модель будущего идеального гражданина вполне себе эквивалентна "новому советскому человеку" который и за станком постоит и посаморазвивается читая "одобренную соцагитку" от политрука, а когда партия позовет и под ружье встанет, а если принять во внимание второе видео, где он дословно цитирует Маркса с его "бытие определяет сознание", да и плановая экономика, на которой держится проект Венера, уже строилась не раз, начиная с опытов иезуитов над индейскими племенами в Новом свете и кончая печально известным большевизмом, так что хоть я сам давно смотрел его видосы и знакомые смотрели и сопереживали, ведь идея свободы угнетенных это больная точка любого общества хорошо возбуждающая эмоции, и в некоторых местах до сих пор считаю их разумными, ведь нужно же разбавлять ложь правдой, но в целом его проект это кристаллизованный технокоммунизм, только вместо рабочих и крестьян роботы, а люди всесторонне развиты и вообще не нужно умирать, об этом можно помечтать, но как перейти от капитализма к проекту В он не говорит, а это самое важное.
Аноним 15/06/18 Птн 03:30:05  440836
1. Насколько максимально быстро может наступить смерть от отравления ртутью (подразумевается более-менее реальный способ, а не телепортация атомов ртути прямо к нейронам)?
2. Если влить ртуть в жидкую сталь, будет пиздец-пиздец пожёщще, чем от воды, или так себе не берём в расчёт отравление местности?
Аноним 15/06/18 Птн 06:35:46  440837
Могут ли мутагенные вещества давать полезные мутации? Или от них только уроды рождаются?
Аноним 15/06/18 Птн 07:06:43  440840
>>440829
Я не знаю как ты так раскрыл скобки, то видео спокойно можно посмотреть, если закрывать просто сообщение о записи. Там доказывается, что независимые случайные величины, у них сумма их, равна сумме их дисперсий.
Ну еще и вспоминаем, что D[cX]=c^2D[X]
>>440829
>выборка по средним величинам значит каждое Xi = μ тут ты не прав. У тебя средняя величина, имеет другое распределение. И ты не можешь просто раскрыть так.

Понятно, что измерения одной и той же величины в разное время, это независимые величины. Поэтому можно легко написать D[сумм(X)/n]=n^-2 сумм(D[X]).
Аноним 15/06/18 Птн 07:58:37  440843
>>440840
Спасибо, вроде въехал
Аноним 15/06/18 Птн 11:47:19  440855
я правильно понял, что мы полностью продукт нашего мозга, а сознание лишь свидетельствует часть его работы?
Аноним 15/06/18 Птн 11:54:08  440856
>>440855
А продуктом чего еще мы можем быть? Торсионых полей? Промысла божьего? Надинфморационного эфира? Всеобщего трансцендентного сознания? Понятно же, что это всего лишь биохимия мозга - нейроны крутятся, разум мутится. Осталось только выяснить детали процесса.
Аноним 15/06/18 Птн 11:58:31  440857
>>440856
меня интересует роль сознания в работе мозга. оно только наблюдает и создает субъективное ощущение я?
Аноним 15/06/18 Птн 12:07:14  440859
smartchimp.webm (1344Кб, 384x288, 00:00:36)
>>440856
>Понятно же, что это всего лишь биохимия тела
Пофиксил редукционистского дурачка.
>>440857
Сознание в основном занимается геноцидом своего носителя и ему подобных. В отсутствие сознания мы мыслим вдесятеро чётче и вдвое быстрее.
Аноним 15/06/18 Птн 12:08:55  440860
>>440859
>В отсутствие сознания мы мыслим вдесятеро чётче и вдвое быстрее
откуда инфа?
Аноним 15/06/18 Птн 12:12:05  440861
>>440860
Из жизни взрослого человека. Чтобы быть гонщиком, лётчиком, снайпером нужно овер9000 часов дрочить рефлексы. Чтобы быть профессионалом в любой профессии, нужно отключать рефлексию и ебашить с обезьяньим лицом. Тем временем не обременённый сознанием чимп на вебм всё это имеет из коробки.
Вывод? Всё очень плохо.
Аноним 15/06/18 Птн 12:14:20  440862
Алсо можешь погуглить успехи приматов в видеоиграх. Там их рефлексы геймеров с 20-летним стажем очень хорошо заметны.
Аноним 15/06/18 Птн 12:14:43  440863
>>440861
>Вывод? Всё очень плохо.
вывод? кто то не читает книжки
Аноним 15/06/18 Птн 12:15:48  440865
>>440857
Остается понять, что такое сознание. Если мы полезем смотреть в разные источники, то наткнемся на кучу наукообразных определений через другие еще более расплывчатые термины ("...отражение действительности независимо от того, на каком уровне оно осуществляется — биологическом или социальном, чувственном или рациональном бла бла бла"). Проще вообще отказаться от этого термина и смотреть на объективные показатели, например, на момент _осознания_ некоего решения, воспринимаемого подопытным, как принятым им самим. Из ряда эксперементов (лень гуглить) удается заключить, что активизация зон мозга, ответственных за принятие решения возникает раньше, чем момент этого самого _осознания_ принятия решения. Интерпретировать это в терминах гуманитарной психологии можно, как угодно, конечно.
Аноним 15/06/18 Птн 12:15:55  440866
>>440863
Вот это ты меня прям уделал.
Аноним 15/06/18 Птн 12:16:56  440867
>>440865
Интерпретация заключается в том, что сознание это паразит.
Аноним 15/06/18 Птн 12:17:12  440868
>>440859
>Сознание в основном занимается геноцидом своего носителя и ему подобных. В отсутствие сознания мы мыслим вдесятеро чётче и вдвое быстрее
>Подразумевая, что у шимпанзе нет сознания
И кто тут редукционный дурачок?
Аноним 15/06/18 Птн 12:18:21  440869
>>440868
Человеческого - нет, разумеется.
Аноним 15/06/18 Птн 12:20:33  440870
>>440865
да про решения это известно, эксперимент либетта или как то так. я понимаю что сейчас у научного сообщества нет ответа на вопрос ЧТО такое сознание, но его роль можно определить, и вроде определили, тут я не в курсе, в общем если это известно, хочу знать
>>440867
>>440869
зачем ты позоришься?
Аноним 15/06/18 Птн 12:24:29  440872
>>440870
>нет ответа на вопрос ЧТО такое сознание
>его роль можно определить
Как это в рамках одно синтаксически корректного предложения получилось то?
Аноним 15/06/18 Птн 12:28:45  440873
>>440872
не вижу противоречия
Аноним 15/06/18 Птн 12:31:34  440874
>>440870
>зачем ты позоришься?
Зачем ты аргументируешь свои возражения?
Ты ёбаный в рот дурачок. У тебя нет определения сознания, но ты свято уверен, что это что-то единое, чего в человеке больше, а в чимпе меньше. Тебе надо мозг от христоза мыть, а не о науке спорить.
Аноним 15/06/18 Птн 12:33:58  440875
>>440874
ты меня путаешь с другим аноном
15/06/18 Птн 15:22:56  440883
Блядь система образования симулякр.
Меня должны были в био реактор кинуть, а я корочку получил.
Все эти тесты бумажки для того чтобы богатый дядя разрешил работать на него.
Чтобы создать иллюзию что рабочих мест хватает.
А знание это просто теория и практика берёшь и учишь и всё.
Аноним 15/06/18 Птн 15:54:12  440887
>>440883
>Все эти тесты бумажки для того чтобы богатый дядя разрешил работать на него.
Нет. Все эти тесты бумажки для того, чтобы ты занят был и по карманам не шарил. И в жопу в подворотне не давал.
Аноним 15/06/18 Птн 16:08:18  440889
>>440823
Волны поглощаются, переизлучаются. В микроволновке продукт и поглощает, поэтому на ней написано - не включать пустой, иначе сам излучатель сгорит.
>Каким вообще образом происходит поглощение волн
читай квантовую электродинамику
Аноним 15/06/18 Птн 17:18:43  440894
>>440865
А почему бы момент активации зон не считать за начало осознанного принятия решения?
15/06/18 Птн 18:15:13  440897
Что такое ум и почему люди с годами не меняются?
Почему
Аноним 15/06/18 Птн 18:23:09  440898
>>440897
Почему я не умнею, моё поведение остаётся прежним как было в 12 лет так и осталось в 21.
Ведь сказано что характер меняется с годами
Аноним 15/06/18 Птн 19:15:16  440900
>>440894
>А почему бы момент активации зон не считать за начало осознанного принятия решения?
Как я выше написал, считать можно всё что угодно, наука в этот момент кончается и начинается философия. Но так, для отправной точки для философии, можно посмотреть на случаи людей с нарушением работы мозга, когда, например, решения принимаются, как обычно, но человек их не воспринимает, как совершенные по своей воле и, наоборот, когда у человека возникает иллюзия, что все вокруг происходит по его воле.
Аноним 15/06/18 Птн 19:19:30  440902
>>440894
Потому что это не осознанное принятие решения, гений.
Аноним 15/06/18 Птн 20:28:46  440905
При давлении всего лишь около 80 мПа воздух будет иметь плотность как у воды? (температура 20 градусов цельсия)
мимоаутист
Аноним 15/06/18 Птн 21:32:31  440908
Правда, что в кв механике частицы размазаны не только в пространстве, но и во времени?
Аноним 15/06/18 Птн 23:17:27  440909
>>440900
Что почитать на эту тему, чтобы вот с твоими примерами, вот это вот всё? Ну и философские размышления посоветуй.
Аноним 15/06/18 Птн 23:20:28  440910
>>440897
Ну как же люди не меняются, если ты пока писал сообщение очень сильно изменился на многих уровнях? По поводу ума, думаю, что тебя интересует другой вопрос и попробуй еще раз.
Аноним 15/06/18 Птн 23:23:31  440911
>>440908
Ну смотри, в уравнение Дирака, время входит на равных с пространственными компонентами.
Аноним 16/06/18 Суб 00:29:40  440913
Где можно узнать про лучевую болезнь и вот это всё?
Аноним 16/06/18 Суб 01:56:18  440915
>>440910
Меня достал бихевиоризм, я же не могу что-то сделать пока не переделаю себя, а человека переделывает опыт и место в жизни.
Это выходит моя судьба предопределена.
Вот я и злой с этого.
И главное я не нахожу то что кто либо может что-то сделать вне его нормы поведения.
Чтобы это человек с опытом не смог предсказать.
А есть опровержение бихевиоризма как абсолютно антинаучной теории?
Что в реальном мире есть место чистому хаусу без логики.
Аноним 16/06/18 Суб 07:31:41  440919
>>440909
крис фрит мозг и душа
Аноним 16/06/18 Суб 08:23:35  440922
>>440915
Получается я родился таким, ведь ничего не может измениться, вот и злой.
Как дерево вырасло не в том месте, так и я не свободнее этого дерево.
Аноним 16/06/18 Суб 09:53:36  440927
>>440915
>Это выходит моя судьба предопределена.
Предопределена чем? Условиями окружающей среды, очевидно же.
А они могут быть динамически изменяющимися.
Пиздуя детерминистично по пути судьбы, в одно точно заданное будущее,
ты выступаешь в роли носителя какой-то инфы в пиздатом суперсистемном суперкомпе,
а в среде, на этот путь истинный - носителя какой-то хуиты,
как-бы наставляют переключатели, ну чтобы эта хуета - пошла куда-то туда и именно туда.

Короче, весь прикол в среде. И в том, может ли существовать какой-либо конструктивный хаос.
То есть, может ли существовать истинный, организованный рандом,
конструктивно изменяющий условия окружающей среды.
Или же среда тупо запрограммирована неким изначально предустановленным псевдорандомом?
Почему конструктивный? Да потому, что обычный хаос - деструктивен,
он он просто вносит помехи, к тому же есть самокорректирующиеся коды, нивелирующие их.
А вот в чистом хаосе и неразберихе - походу, просто работают - законы самоорганизации.
Это синергетика. Там - ячейки Бенара. https://www.youtube.com/watch?v=H-9do9iayyc
По всей видимости, так появляются и исчезают соборные интеллекты, империи всякие,
союзы, общества, а также поглощаются друг-другом - различные корпорации.

Но и это ещё не всё... Если истинного рандома не существует,
либо если он есть, но он не может конструктивно организовываться в информационную систему управления,
индетерминизирующую процессы самоорганизации в среде,
то всё изначально предопределено, то ни о каком хаосе речи идти не может в принципе.
Тогда, получается тупо - одна сплошная судьба.
Аноним 16/06/18 Суб 10:05:57  440928
Прочитал про опыт с червями, когда одного червя, прошедшего лабиринт, скармливали другому, тот как-то воспринимал знания и тоже проходил лабиринт. Как такое возможно?
Аноним 16/06/18 Суб 10:10:29  440929
>>440927
>Предопределена чем? Условиями окружающей среды, очевидно же.
А ещё, судьба может быть предопределена самой внутренней структурой.
То есть, в процессе действия информационной системы управления,
самоорганизующейся конструктивно из какого-либо хаоса и/или рандома,
эта инфосистема не может изменить внутреннюю структуру объекта управления,
рассматривая его субъективнейшим образом - как целостный объект, юнит.
Таким образом, внутренняя структура соответствует предназначению объекта управления,
либо указывает на принципиальную неспособность
(то есть нецелесообразность, невозможность и/или нежелание)
управляющей инфосистемы - изменить её и настроить эту структуру, правильно,
то есть в строгом соответствии с её предназначением.
И ей приходится мириться, или надеяться на процессы сампроизвольной реструктуразиации
этих ебучих структур.
Аноним 16/06/18 Суб 10:24:14  440930
>>440928
Не черви, а планарии.
На, вот тут почитай:
https://ru.wikipedia.org/wiki/Перенос_памяти#Опыты_Мак-Коннелла
Там регенерация через РНК, куда сохраняется инфа.
Аноним 16/06/18 Суб 22:37:14  440983
Почему скорость света - это прямо какой-то сакральное понятие? Типа её нельзя превысить, а если превысить ,что произойдёт прямо-таки что-то невообразимое.
Хули там произойдёт-то? Ну превысишь, будешь во тьме лететь, свет останется сзади. Со звуком же так и происходит, чё тут прямо такого охуенного может произойти?
Аноним 16/06/18 Суб 23:31:46  440986
>>440983
>Почему скорость света - это прямо какой-то сакральное понятие?
потому что внезапно все твое существование завязано на скорости света
Аноним 16/06/18 Суб 23:43:16  440987
>>440986
Также как и скорость звука в 40-х, лол.
Аноним 16/06/18 Суб 23:53:11  440988
>>440987
дебич не понимает разницы между макроскопическим явлением и фундаментальным
Аноним 17/06/18 Вск 00:04:19  440989
>>440983
Все частицы движутся с одной скоростью - со скоростью света и для них не существует ни времени, ни расстояний, некоторые из них, взаимодействуя с полем Хиггса, испытывают циклы хиральных превращений, которые превращают безмассовую, безвременную частицу в динамически существующую частицу, имеющую хиггсовскую массу и временную шкалу.
Т.е. дело не в том, что частицу нельзя разогнать выше скорости света, а в том, что на деле нет таких частиц, которые бы двигались с какой-то отличной от с скоростью, все наблюдаемые скорости массивных частиц - фикция, это всё тени настоящих безмассовых частиц, которые имитируют массу, скорость и время в классическом представлении.
Аноним 17/06/18 Вск 00:10:45  440990
Вопрос по квантовой механике. В чём отличие смешанного состояния от суперпозиции чистых состояний?
Аноним 17/06/18 Вск 00:25:47  440992
>>440989
>взаимодействуя с полем Хиггса, испытывают циклы хиральных превращений
Воу-воу, я же быдло, я слов-то таких не знаю. Ты давай попроще, поближе к народу.
Аноним 17/06/18 Вск 00:26:45  440993
>>440988
Фундаментальное оно пока не преодолено. А ты просто ретроград.
Аноним 17/06/18 Вск 01:01:41  440994
>>440989
Т.е. я правильно понимаю, что все частицы - это такие как бы СОЛИТОНЫ в поле Хиггса (привет кефирщикам, ага), и что скорость света можно превысить, если каким-то образом создать в поле Хиггса как бы ударную волну?
Аноним 17/06/18 Вск 06:49:06  440997
А есть в теории мультивселенных какие-нибудь конструктивные рассуждения о том, какие вселенные могут существовать, а какие нет? Например, может существовать вселенная, в которой люди вместо рукопожатия ебут друг друга в жп и наоборот, а остальное всё абсолютно так же? Или где непрерывно орут с громкостью 500дб, но всем похуй при этом. Должны же быть какие-то ограничительные условия.
Аноним 17/06/18 Вск 07:27:29  440998
>>440983
Потому что это скорее геометрия пространства, нежели ограничение самой скорости.
Нет, её конечно можно превышать, но локально видимо нет, только если во всяких пузырях
Аноним 17/06/18 Вск 07:31:36  440999
>>440990
Смешанное, это когда у тебя например электрон ТОЧНО какой-то спин имеет, но ты не знаешь, и только вероятности прикидываешь. В том смысле, что он обладает определенным спином еще до измерения. Считай что это как-бы статистическая вещь, как распределение по скоростям молекул и т.п. А вот суперпозиция, это именно сумма, у которой нет выделенного спина до измерения.
Аноним 17/06/18 Вск 10:33:02  441005
>>440997
>Должны же быть какие-то ограничительные условия.
Гугли "тонкая настройка". Ограничительные условия - соотношения фундаментальных констант в узких пределах, при которых всё не ёбнет. А во что там сложатся белковые зверушки - всем похуй.
Аноним 17/06/18 Вск 11:53:42  441009
>>440994
Нет, всё движется со скоростью света, даже небо и Кефир.
Аноним 17/06/18 Вск 12:14:14  441012
Область Вернике отвечает только за речь? Или и за письмо тоже? Тексты устаревшие, дайте свежий на эту тему.
Аноним 17/06/18 Вск 12:36:26  441013
>>441009
Кефир быстрее света, кефирные взаимодействия ограничены лишь упругостью конкретной области пространства-времени.
Аноним 17/06/18 Вск 12:45:23  441014
>>441013
Кефира не существует.
Аноним 17/06/18 Вск 13:05:20  441018
>>441014
Тебя не существует.
Аноним 17/06/18 Вск 13:25:08  441021
Привет. Одна бестолочь насыпала хлорки в некоторых местах возле дома, при открытых окнах конкретно так тянет, пришлось закрыть. Сразу вызывать мчс, чтобы залили всё водой или же на свежем воздухе пары хлора выветрятся сами? Само собой, там не 10кг, но всё же, не хотелось бы словить какое-нибудь хроническое отравление.
Аноним 17/06/18 Вск 13:35:25  441022
>>441021
Лучше таки позвонить в это самое МЧС и задать вопрос там.
Аноним 17/06/18 Вск 15:36:34  441027
Если Вселенная расширяется из некоей точки, то значит в этой точке энтропия была равна 0, потом произошел Большой Взрыв но энтропия по прежнему равна нулю, потому что в противовес
хаосу начали образовываться планеты, которые уравновешивают разбегание частиц, значит ли это что феномен жизни, т.е. первая клетка, возможно возник, как попытка Вселенной уравновесить взрыв какой-нибудь огромной звезды и повышение энтропии?
Аноним 17/06/18 Вск 16:20:33  441029
>>441027
Иди лучше бухни что-ли.
Аноним 17/06/18 Вск 16:22:30  441030
>>441027
Она расширяется не из некой точки
Аноним 18/06/18 Пнд 13:54:09  441106
>>441030
И не из точки, и не значит, что энтропия равна 0, и не планеты, и не в противовес хаосу, и не уравновешивают, и не значит. что феномен жизни, и жизнь возникла еще до первой клетки, и не уравновесить взрыв звезды. В общем чувак где-то баззвордов нахватался, дернул поллитры сверху и понеслось.
Аноним 18/06/18 Пнд 14:31:32  441107
Сап двощ. Пересмотрел недавно парк юрского периода, любимый фильм детства, вся херня. Всегда любил динозавров и вот во мне снова проснулся интерес с еще большей силой. Хочу почитать что нибудь по сабжу, только не всратые детские энциклопедии а более научную и углубленную литературу. Чтобы всё в подробностях и тд. Реквестирую годных книг по динозаврам.
Аноним 18/06/18 Пнд 14:41:52  441108
Почему у китов такой огромный мозг? Они ведь не на много умнее шимпанзе, но весит он 9 килограмм, а не 900г. Зачем он ему? Ведь гораздо выгоднее использовать свободное пространство для жира, чем для хуйни, которую можно сжать в 10 раз.
мда, как такая хуйня вообще пришла мне в голову перед сном
Аноним 18/06/18 Пнд 14:43:10  441109
>>441107
Википедии недостаточно?
Аноним 18/06/18 Пнд 14:48:12  441110
>>441109
Хотелось бы что то основательное. Просто такое чувство что на википедии всё кусками из тех же детских энциклопедий.
Аноним 18/06/18 Пнд 14:56:17  441111
>>441110
Список литературы не для тебя?
Аноним 18/06/18 Пнд 16:04:41  441114
>>439790 (OP)
Привет, антоши. Я читал Стелазина и наткнулся на такую штуку как электромагнитное подавление зоны TPJ, т.е. электромагнитное подавление зоны мозга, не могли бы вы объяснить как это делается, что то в голову лезет всякая фантастика, а беглый гуглеж не помог.
Аноним 18/06/18 Пнд 16:53:14  441117
Привет всесторонне развитый анон. Вопрос не в тему. Вот уволили человека по статье, с работы, с крупного предприятия и говорят щас типо хуй те а не работа. В серьезных конторах не возьмут. Все в какойто там базе. Поясните как это устроено чеза базы итп. (Реч о серьезных работадателях -топ 30 рф)
Аноним 18/06/18 Пнд 17:17:07  441119
Накидайте плз список компаний, основанных физиками! Я сам же и начну: Fairchild semiconductor, Intel.
Аноним 18/06/18 Пнд 18:07:57  441120
cirno.jpg (114Кб, 535x750)
Зачем весь мир тотально лжет насчёт вырождения от имбридинга?

Ведь достаточно даже базовых школьных знаний генетики и законов Менделя, чтобы всё опровергнуть.
Для чистоты эксперимента у нас есть Х-хорошие доминантные гены и п-плохие рецессивные. Для чистоты эксперимента берём 8 потомков:
Хп + Хп = 2Хх, 4Хп, 2Пп - последние выбраковываются из потомства и именно из-за этих 25% брака вся паника.

Остаются 4Хп, чьё скрещивание даёт тот же результат, что и выше, с 25% брака, и 2Хх - два потомка с "чистыми" генами, чьё потомство не будет иметь дефектов в любом случае:
Хх + Хп = 4Хх, 4Хп - продолжим, чтобы оценить второе поколение:

33%, что пары распределятся так: Хх + Хх, 2(Хп + Хп) с потомством:
8Хх, 2(2Хх, 4Хп, 2Пп) - суммируем: 12Хх, 8Хп, 4Пп.

и 66%, что распределятся так: 2(Хх + Хп), Хп + Хп с потомством:
2(4Хх, 4Хп), 2Хх, 4Хп, 2Пп - суммируем: 10Хх, 12Хп, 2Пп - умножаем на два за счёт соотношения шансов (66%/33%): 20Хх, 24Хп, 4Пп.

Суммируем оба варианта:
32Хх, 32Хп, 8Пп - выбраковываем Пп и видим, что количество "чистых" потомков возрастает, а доля "брака" сокращается с 25% в первом поколении до 1/9=11,(1)% уже во втором поколении! И далее тенденция сохраняется, позволяя уменьшать долю носителей "брака" в рецессивных генах, пока их немногих носителей не порешает случай.

Итак, вопреки устоявшемуся мнению, имбридинг имеет тенденцию не к росту % "брака", а к затуханию, но нам лгут, что целые цивилизации (египетских жрецов, например) вымерли от имбридинга. Зочем?
Аноним 18/06/18 Пнд 22:53:48  441139
image.png (261Кб, 485x787)
image.png (277Кб, 712x938)
>>441107
Можешь покопаться в b-ok.xyz/s/?q=dinosaur&e=1&yearFrom=2012&yearTo=2018&language=&extension=&t=0
Там даже есть про копролиты динозавров на Аляске.
Аноним 18/06/18 Пнд 23:09:09  441143
Один советский учёный кажется Капица утверждал, что после новогоднего бокала шампанского его креативность он как-то по-другому сказал, но похоже восстанавливается только через месяц. Являются ли его личные ощущения скорее самовнушением, или даже малые дозы алкоголя действительно могут отуплять на месяц?
Аноним 18/06/18 Пнд 23:53:35  441146
>>441143
Под воздействием спирта эритроциты крови склеиваются между собой, что способствует образованию тромбов.
Сгустки крови способны перекрыть просвет артерии и лишить ткани питания и кислорода, что приведет к их гибели.
https://alkogolik-info.ru/vliyanie-na-zdorove/obshaya-zdr/alkogol-i-tromby.html#vliyanie-etilovogo-spirta-na-krovenosnuyu-sistemu

Какого хрена там скопировать нельзя? Вытащил из HTML-кода.
Аноним 18/06/18 Пнд 23:56:50  441147
>>441143
>советский учёный
>что-то утверждал на основании "приблизительно так ощущаю" вместо исследований и экспериментов
>в 21м веке кто-то спрашивает, достаточно ли авторитетно ощущение авторитетного (но это не точно) учёного?
Все известные мне научные факты говорят, что учёный пиздел, как восьмиклассница, и просто нихрена не хотел или работать или бухать со всяким быдлом.
Вот тебе калькулятор на всякий случай:
https://alcofan.com/kalkulyator-alkogolya
Аноним 19/06/18 Втр 00:00:39  441148
>>441146
Тромб это тромб. Никакого "восстановления через месяц" не происходит. Даже если предполагать, что гибкая структура мозга через месяц смогла построить цепочки вокруг уничтоженных клеток, это всё-равно необратимая деградация.

Ну а если бы от бокала газированного винца в мозгу начинался бы гроб-пидор-кладбище с тромбами и отмиранием тканией, то пьющих учёных бы не существовало в принципе.
Аноним 19/06/18 Втр 00:02:31  441149
>>441148
При чём тут пьющие учёные, школец? Умные люди по статистике пьют больше медианных, но меньше дебилов.
Аноним 19/06/18 Втр 00:29:36  441150
>>441108
Потому что у него очень жирная туша и ею надо как-то управлять. Собственно его мозг в основном состоит из отделов отвечающих за передачу и обработку нервных импульсов к мышцам.
Аноним 19/06/18 Втр 00:29:57  441151
>>441149
Ты там стекломоя навернул и ослеп?
Аноним 19/06/18 Втр 00:41:50  441152
>>441151
Нет, я вижу, что ты как бы пытался сделать противопоставление "учёный - умный, неучёный - алкаш". Я тебя просветил, что получилось не очень.
Аноним 19/06/18 Втр 05:12:33  441155
>>441152
>я вижу, что ты как бы пытался сделать противопоставление "учёный - умный, неучёный - алкаш"
Это прекрасно, но, вообще-то, я не пытался и ничего подобного у меня не написано.
Аноним 19/06/18 Втр 08:39:47  441160
>>441119
Нет, ты и дальше так и будешь работать за 13к, жить у мамки, или вообще целиком у неё на шее за её счёт.
Аноним 19/06/18 Втр 09:42:26  441162
чтобы подробно изучить мозг, нужно ли мне ебаться с учебниками по биологии времен средней школы и учить анатомию и етц всякую хуйню?
Аноним 19/06/18 Втр 10:47:05  441165
Сап, зачем учиться если интеллект врожденное?
Аноним 19/06/18 Втр 10:55:03  441166
>>441162
нет, достаточно мемасиков с двощей
Аноним 19/06/18 Втр 11:04:51  441167
>>441166
не люблю мемы
Аноним 19/06/18 Втр 12:09:45  441171
Каким образом глюкоза превращается в энергию? Я знаю что это очень сложный процесс, и что глюкоза еще распадается на всякие кислоты, аденозинтрифосфаты но в конечном счете какое конечный элемет этих преобразований из которого и получается энергия. Вообщем какое вещество является последним элементом который и дает нам энергию в клетки и каким образом эта энергия выделяется, в виде чего?
Аноним 19/06/18 Втр 12:15:03  441172
>>441162
А зачем тебе, если не секрет?
Аноним 19/06/18 Втр 12:30:37  441174
>>441171
Переносит энергию у нас ATФ. При расщеплении глюкозы пару штук получается из AДФ.
При синтезе каких-нибудь энергозатратных белков или веществ, AТФ тратится и превращается обратно в AДФ.
Аноним 19/06/18 Втр 12:33:55  441175
>>441172
вкатываюсь в философию сознания
Аноним 19/06/18 Втр 12:34:33  441176
>>441172
ну и вообще хочу понять что я из себя предствавляю
Аноним 19/06/18 Втр 12:35:03  441177
>>441172
в биологическом смысле
Аноним 19/06/18 Втр 12:37:06  441178
>>441175
Для философии биология не нужна. Делается (всегда делалось) это так: придумываешь сверхценную идею и толкаешь ее везде.
Аноним 19/06/18 Втр 12:38:18  441179
>>441178
годная теория должна быть связана с биологией в этом вопросе
Аноним 19/06/18 Втр 12:46:01  441182
>>441179
Да не, они все равно(не)годные. Из биологии только умных слов добавить можно для сокрытия смысла. Короч, смотри, тут выше писали про то, что решения принимаются до их осознания. К черту биологию, к черту исследования, этого тебе уже достаточно (если что, в википедии можно ссылок поднатаскать).
Можно делать на этой основе любые философские выводы, например:
1) Человек - это биоавтомат, детерменированный и безвольный, а поэтому личностью не являющийся и не несущий ответсвенности за свое поведение
2) Сущность человека определяется высшими имманентными коннотации транфсфигурального поля, поэтому личность существует, но не определяется бренным телом
3) Религия, но это уже придумали до тебя.
Аноним 19/06/18 Втр 13:10:55  441186
>>441182
>Можно делать на этой основе любые философские выводы
эти выводы не будут иметь достаточной базы аргументов для подтверждения, или вообще чтобы иметь хоть какую то ценность. я еще не вкатывался туда и уже понял, что все намного сложнее.
>К черту биологию, к черту исследования, этого тебе уже достаточно (если что, в википедии можно ссылок поднатаскать).
не пиши мне, если что
Аноним 19/06/18 Втр 13:21:54  441187
>>441186
>не будут иметь достаточной базы аргументов
Ты много от философии хочешь.
>не пиши мне, если что
Правильно. Если засунуть голову в песок, то манямирок вновь станет розовым. Удачи.
Аноним 19/06/18 Втр 13:59:26  441189
>>441178
>Делается (всегда делалось) это так: придумываешь сверхценную идею и толкаешь ее везде.
Попытался наехать на философию, получилось про науку.
Аноним 19/06/18 Втр 14:13:27  441190
>>441189
С наукой сложнее немного, любую идеи опыт может разъебать, как с гомеопатией например.
Аноним 19/06/18 Втр 14:22:26  441191
jewofanimes.jpg (2459Кб, 2568x3000)
>>441190
Только опыт делают 10 разных лаб, и у всех получаются разные результаты, а потом мы выбираем ту лабу, в которой получились интересующие нас.
Аноним 19/06/18 Втр 14:31:52  441193
>>441191
Ты же далек от реальных лаб? Я правильно понял?
Аноним 19/06/18 Втр 14:35:10  441195
>>441193
Примерно так же далёк, как ты от философии. Но это не мешает нам с тобой пронзительно визжать свои сверхценные мнения, не так ли.
Аноним 19/06/18 Втр 14:42:04  441196
>>441174
Можешь простыми словами объяснить что такое АДФ И АТФ и каким образом это вещество превращается в энергию.
Аноним 19/06/18 Втр 14:43:10  441197
>>441195
Этот диалог тут уже происходил кстати. Я вообще другой анон если что. Про философию ничего не писал.
Аноним 19/06/18 Втр 14:49:34  441198
>>441193
ОК, давай попробуем:
1. Человек есть не что иное, как ряд его поступков.
2. Человек не только ряд его поступков, но и его нравственные убеждения, которыми он руководствовался, совершая их.
Одно из них - утверждение великого философа, другое я только что на коленке сочинил.
Вопросы: ладно хуй с ним, иди гугли, где тут цитата, а где хуйня на постном масле, но только ответь на вопрос: чем одна фраза достовернее другой или какими там философы критериями меряются?
Аноним 19/06/18 Втр 14:53:42  441199
>>441198
1. А = Б
2. А = Б = В
Одно из этих утверждений я извлёк из научного издания, второе сочинил только что сам. На основании этой вырванной из контекста поебени я пытаюсь задать какой-то вопрос, но только с треском пачкаю себе штанцы.
Аноним 19/06/18 Втр 14:55:44  441200
>>441198
Первая фраза явно пафоснее, поэтому она скорее не твоя. Проблема обоих высказываний, что мне нужны четкие определения "человек" "есть" "поступок" "нравственные убеждения". Чтобы я как-то мог их сопоставить.
Философия она же немного о другом, она всегда в отрыве от материального.
Аноним 19/06/18 Втр 15:02:29  441201
>>441200
>всегда в отрыве от материального
>Подразумевая, что есть что-то, кроме материального
Дроаконоведение тоже в отрыве от материального. Любую хуйню придумай, не имеющую предмета исследования, вот тебе и кандидат в новую философию.
>>441199
>вырванной из контекста поебени
Давай, приведи мне весь этот гегелевский шизофазнический контекст, где он за грудой сложносочиненных предложений дай бог выдает супер-банальности, а чаще всего просто противоречит сам себе.
Единственное место, где философия хоть что-то могла предложить рационального - это когда она взялась определить чем наука отличается от не науки, да и то философией это направление мысли называть - только примазываться.
Аноним 19/06/18 Втр 15:08:31  441204
>>441196
Ну оно как бы несет в себе энергию. Как батарейка.
Если тебе нужно будет сделать в организме какую-то реакцию, где понадобится излишек энергии "например синтез глюкозы", то тебе поможет AТФ, фермент его тоже возьмет в реакцию, выпустит AДФ, а она уже спокойно пойдет.
Ну еще там две молекулы НАДH получаются из глюкозы, но они считай носят электроны, одна молекула два электрона носит и является довольно сильным восстановителем.
Аноним 19/06/18 Втр 15:08:53  441205
>>441201
P.S. Ну и вот вам еще гегелевской классики:
С постигнутой им мыслью, что единичное сознание в себе есть абсолютная сущность, сознание уходит обратно в само себя. Для несчастного сознания в-себе-бытие есть потустороннее его самого. Но движение несчастного сознания завершило в нем то, что единичность в ее полном развитии или единичность, которая есть действительное сознание, была установлена как негативное его самого, т.е. как предметная крайность или, иными словами, его для-себя-бытие было исторгнуто из себя и превращено в бытие

Срочно нужен КОНТЕКСТ. Без КОНТЕКСТА в эту поебень, конечно, въехать не представляется возможным.
Аноним 19/06/18 Втр 15:29:05  441206
Что почитать про любые интересные циклы по хардкору. Вот у Земли есть цикл оборота вокруг солнца, дальше оборот вокруг оси, есть цикл жизни помидоров и "женский цикл. Есть даже общие черты в циклах жизни разных империй, научных представлений о мире и идеологий. Есть циклы жизни грибов и вирусов в человеке, как и течение шизы например. Думаю эта тема очень интересно раскрывается даже в неожиданных предметах изучения. Например сердечники, родившиеся в ноябре чаще умирают в сентябре. Или россияне миллениалы и пгт скорее всего покончат с собой в 49 лет. Или если раз в 15000 лет появляется болезнь, которая вырубает большинство людей, то когда следующая вспышка? Ну вы поняли.
Аноним 19/06/18 Втр 15:30:40  441207
>>441201
>Дроаконоведение тоже в отрыве от материального. Любую хуйню придумай, не имеющую предмета исследования, вот тебе и кандидат в новую философию.
И математика ещё.
Аноним 19/06/18 Втр 15:34:25  441208
>>441206
из пгт
фикс
Аноним 19/06/18 Втр 16:04:49  441209
Зачем нужны гуманитарные науки если придумали Бихевиоризм , психология, философия.
>>440929
Это удивительно оказывается наука и творчество это накопленный опыт рефернтовhttps://youtu.be/-IZ7TONVK24 я всегда так думал что человек биоробот.
Жак Фреско подтвердил что среда обитание и воспитание формирует поведение
и характер человека.
А я от себя добавил, когда человек согласится что он биоробот, все гуманитарные науки будут не нужны в образовании, ведь будет бихевиоризм с единым языком.
Аноним 19/06/18 Втр 16:30:45  441210
>>441206
https://en.wikipedia.org/wiki/List_of_cycles
Аноним 19/06/18 Втр 17:09:28  441211
я правильно понял, что размер миндалевидного тела положительно коррелирует со склонностью человека испытывать страх?
Аноним 19/06/18 Втр 17:20:51  441213
>>441176
ГОВ НО
Аноним 19/06/18 Втр 17:37:00  441214
>>441209
>я всегда так думал что человек биоробот
Биоробот - это конструкция, созданная человеком. Люди, очевидно, не могут ещё создавать ни себе подобных, ни даже самой простейшей жизни без естественного начала процесса. Поэтому все кудахты о "всего лишь наука" оставь на пятое тысячелетие от РХ, а до тех пор соси хуй и крути болты.
Аноним 19/06/18 Втр 19:16:16  441224
>>441114
бамп вопросу. Или скажите куда с ним пойти.
Аноним 19/06/18 Втр 22:15:28  441235
Т-рищи физики, вопрос такой назрел. Скажите, пруфаните. Сопротивление какого воздуха больше: Холодного или горячего?. Допустим орудие совершило 2 выстрела абсолютно одинаковыми снарядами. Первый был выпущен в среде с температурой +500 Цельсия, второй при температуре среды -200 цельсия. Начальная скорость снаряда в обоих случаях равна =1600м\с. Скорость какого снаряда через 2 км будет выше? Интересует именно зависимость от температуры среды.
Аноним 20/06/18 Срд 00:19:19  441243
>>439790 (OP)
Почему , например, железо приобретая кристалическую структуру - остается поастичным в какой-то степени,
а , например, кальций в кристалической структуре в смысле - мрамор чрезвычайно не пластичен?

От чего это зависит и какие термины гуглить, что бы понять

Аноним 20/06/18 Срд 00:21:20  441244
>>441243
>пластичным
Фастфикс
Аноним 20/06/18 Срд 00:47:05  441245
>>441235
тут очевидная разница в давлении
Аноним 20/06/18 Срд 00:50:43  441246
>>441243
Ну тут вся разница в самых хим. связях. У тебя в железе металлическая. А в карбонате кальция практически чисто ионная.
Аноним 20/06/18 Срд 01:02:40  441247
IMG3952.JPG (108Кб, 807x453)
>>441246
О, то что надо. Граци. держи няшу
А можешь еще на пальцах обьяснить, почему металическая тянется , а ионная сразу рвется?
Аноним 20/06/18 Срд 01:09:42  441248
>>441247
Всё дело в энергии решетки. Если ты будешь двигать атомы металла, то она не будет сильно расти. Так как атомы металла притягиваются друг к другу как бы равномерно что-ли... Ну то есть куда ты не прилепи атом, он будет тянуться к другим. А а ионов есть анионы и катионы, которые очевидно и притягиваются друг к другу, и отталкиваются друг от друга, поэтому для них нарушение порядка это пиздец критично. Дырку они еще могут потерпеть, а вот если ряд съедет вбок, пиздец будет. Посуди сам
У тебя было в одномерном случае двух линий
+ - + - + - + - + -
- + - + - + - + - +
У тебя разные заряды поближе, а одинаковые подальше.
А теперь смести один ряд относительно другого.
+ - + - + - + - + -
+ - + - + - + - + -
Смотри какой пиздец случился, ряды пытаются оттолкнуться друг от друга, считай ты их расколол.
Аноним 20/06/18 Срд 14:02:10  441268
>>441248
Тоесть все дело в поляризации молекул? Но есть еще металлы разной хрупкости, пластичности и упругости. Это тоже зависит от количества положительных ионов?
Аноним 20/06/18 Срд 14:37:02  441270
>>441268
>поляризации молекул?
Ну в карбонате кальция молекул нет. Там чисто ионы.
>Но есть еще металлы разной хрупкости, пластичности и упругости.
Металлы не только металическую связь могут проявлять. Но и ковалентные связи образовывать друг с другом. Понятно, что это сделает решетку жестче намного. Пример практически чисто металлической связи это какой-нибудь цезий. Он мягкий и пластичный.
Аноним 20/06/18 Срд 18:14:19  441281
>>439790 (OP)
Потенциал в центра полусферы равен or/(2eps), где o-поверхностная плотность, r-радиус шара. Напряжённость поля есть минус градиент потенциала. Тогда для полусферы напряжённость в центра должна быть равна нулю? Но по чему она не равна?
Аноним 20/06/18 Срд 20:42:38  441287
Здравствуйте!
Будьте добры, объясните, как влияет толщина пластинки в двухщелевом эксперименте С ЭЛЕКТРОНАМИ!!! на интерференционную картину?
Во многих найденных мною описаниях внимания этому вопросу не уделяется.
Или, быть может, киньте линк на подробное описание оригинального экспериментаС ЭЛЕКТРОНАМИ!!11! БЛЯТЬ.

Заранее благодарю.
Аноним 20/06/18 Срд 22:27:17  441292
1308017275001.jpg (5Кб, 180x200)
Если у света нет массы, что и позволяет ему перемещаться со скоростью света. То почему на него действует гравитация чёрных дыр?
Аноним 20/06/18 Срд 22:58:03  441293
>>441292
Потому что гравитация действует на энергию а не на массу.
Аноним 21/06/18 Чтв 00:56:52  441297
>>441287
толщина какой пластинки?
Аноним 21/06/18 Чтв 01:00:17  441298
>>441292
говорят там пространство кривое
Аноним 21/06/18 Чтв 08:00:02  441304
Каким способом легче всего сохранить свой ДНК лет на 10?
Аноним 21/06/18 Чтв 08:01:22  441305
>>441304
ДНК материал, вне тела, конечно же
фикс
Аноним 21/06/18 Чтв 08:18:44  441306
>>441304
Любым. Ну если хочешь заморочки, то в пакетик темный, не пропускающий уф лучи. Туда положи волосню, кожу с пальца ноги содри
Аноним 21/06/18 Чтв 10:42:09  441308
>>441304
телку себе найди наконец
Аноним 21/06/18 Чтв 10:45:13  441309
>A recent study based on molecular simulations and compliance constants describes molecular recognition as a phenomenon of organisation.[18] Even for small molecules like carbohydrates, the recognition process can not be predicted or designed even assuming that each individual hydrogen bond's strength is exactly known.
Чиво, блядь? То есть если я воткну пен в эпл и кину с балкона, оно наверх полетит что ли? Нет, оно вниз полетит, потому что и на пен и на эпл действует гравитация и я могу это расчитать. Тогда что эта собака несёт, кто его вообще пропустил в журнал с такой ахинеей?
http://pubs.rsc.org/en/Content/ArticleLanding/2011/CP/c1cp20097f#!divAbstract
Аноним 21/06/18 Чтв 12:06:52  441314
>>441304
>Каким способом легче всего сохранить свой ДНК лет на 10?
Сделай несколько детей и научи их защищать себя.
Аноним 21/06/18 Чтв 13:27:25  441318
Если человек переместится на 100 млн лет в прошлое, каким для него будут земные условия? Кроме фауны, конечно, лол. Наверняка там были свои особенности. Можно ли дышать воздухом, который был на планете сотни миллионов лет назад?
Аноним 21/06/18 Чтв 13:33:38  441319
>>441297
Разумеется толщина пластинки с щелями.
Аноним 21/06/18 Чтв 14:58:17  441322
darkspot[1].png (18Кб, 600x275)
>>441287
щель должна быть ссужающийся на пик один из вариантов
Аноним 21/06/18 Чтв 15:01:56  441323
>>441322
Кинь линк, плиз, на вариант эксперимента, откуда пикрелейтед взят. Гугл не находит.
Аноним 21/06/18 Чтв 15:24:51  441324
>>441323
>куда пикрелейтед взят. Гугл не находит.
http://studbooks.net/2576316/tovarovedenie/modernizatsiya_sistemy_avtomatizatsii_tseha_osushki_gaza_ukpg_putem_podbora_preobrazovatelya_temperatury_rosy
классика https://youtu.be/LSwCCtvbSdg
Аноним 21/06/18 Чтв 15:30:26  441325
>>441318
Однохуйственными. Можно.
Аноним 21/06/18 Чтв 18:36:15  441329
>>441306
Насколько хорошо сохранится?
Аноним 21/06/18 Чтв 18:45:30  441330
15247749461800.mp4 (634Кб, 1280x720, 00:00:04)
Печень объект, мозг – субъект?
Огненные стрелы = огнестрельное оружие?
Хлопья быстрорастворимой картошки – мучной продукт?
Аноним 21/06/18 Чтв 19:59:52  441332
1529586065-56ba[...].jpeg (66Кб, 700x623)
>>441330
Софистика конструктивное наебалово
Аноним 21/06/18 Чтв 23:33:04  441336
Как мужчина может кормить грудью?
Аноним 22/06/18 Птн 03:41:40  441338
>>441336
1. Если грудь не его
2. Если грудь предварительно накачать молоком
3. Если прикажет партия
Аноним 22/06/18 Птн 13:56:02  441353
>>441336
Если будет колоть пролактин.
Аноним 22/06/18 Птн 14:01:42  441354
>>439790 (OP)
Нахуя в дренажные колодцы насыпают щебень? Как он спасает от заиливания?
Аноним 22/06/18 Птн 16:49:55  441359
Привет. Я к вам с вопросом (естественно, глупым)

С чего начать учить физику, если в школе вообще ничего не знал, вплоть до 11 класса? Серьёзно, я не знаю по физике вообще ничего.

С каких учебников начать, или, может лучше с курсов?

Цель - без зубрёжки научиться именно соображать в физике, самому выводить формулы, и больше практики, больше решения задач.
22/06/18 Птн 20:38:09  441371
Что такое квантовая физика?
Что наблюдатель влияет на реальность?
Что материя может меняться с разной вероятностью?
Аноним 22/06/18 Птн 22:02:45  441375
>>441371
Классика процесс обратим при воздействии наблюдателем
Релятивистская процесс моделирует наблюдатель
Квантовая наблюдатель есть процесс
Информационная процесс это технология
имхо
Аноним 22/06/18 Птн 23:29:47  441382
Почему когда я выпью пива, то гораздо больше устаю от нагрузок?
Аноним 22/06/18 Птн 23:31:43  441383
>>441382
Слабак
Аноним 22/06/18 Птн 23:43:46  441385
Если ускорить скорость движение то время пойдёт быстрее?
Космонавты на станции быстрее стареют чем другие люди?
Аноним 23/06/18 Суб 00:01:59  441387
>>441385
МКС находится высоко над поверхностью и летит со скоростью 27 576 км/ч. относительно Земли, поэтому ее собственное время, во-первых, течет быстрее нашего собственного времени, так как на высоте пространство-время меньше искривлено под воздействием гравитационного поля Земли. И, во-вторых, время на борту замедляется относительно нашего земного из-за высокой скорости космической станции. Осталось лишь вычислить оба этих эффекта и понять, какой из них имеет большее влияние.

К счастью, все подсчеты уже сделаны, и мы можем просто привести конечные результаты. Получается, что из-за меньшего воздействия гравитации время на МКС каждый день начинает идти быстрее на 0,00000354 сек. Замедление же времени, вызванное скоростью МКС, составляет -0,00003 секунды в день. Сложив два значения, мы получим замедление времени, равное -0,00002646 сек. в день.

Очевидно, что полет на МКС — не самый лучший способ продлить себе жизнь относительно других людей: ведь чтобы выиграть одну секунду, вам придется провести на орбите около 100 лет.
Аноним 23/06/18 Суб 00:08:59  441388
>>441387
Там мудак в 60х обламал весь срач, тк малекулярным структурам коллебания похуй в коких системах их смотрят
Аноним 23/06/18 Суб 09:29:11  441401
Какова должна быть эквивалентная доза радиации киллограмма мышечной ткани, чтобы половина клеток в ней с гарантией сдохло?
Аноним 24/06/18 Вск 01:25:05  441467
>>441401
Как быстро сдохло? Просто если у тебя резко 20% клеток сдохнет, то со временем остальные тоже сдохнут из-за отравления продуктами распада тех что сдохли раньше. В общем процесс нелинейный. Или клеток сдохнет мало, но потом рак вытеснит оставшиеся.
Обычно замеряют LD50 - дозу при которой 50% облученных животных сдохло, а клетки отдельно рассматривать тупо, тем более каких клеток, у разных тканей разная чувствительность к облучению.
Аноним 24/06/18 Вск 02:26:17  441469
>>441401
Ионизирующее излучение повреждает ДНК посредством ионизации атомов в нуклеотидах. При этом, жизнеспособность повреждённой клетки может гарантироваться за счёт наличия РНК в цитоплазме, а не за счёт ДНК.
Если часть клеток не умирает от радиации, они повреждаются, что зачастую очевидно уже после процесса митоза у них.
Аноним 24/06/18 Вск 06:09:04  441480
>>441467
Просто мне интересно, какой должна быть мощность луча нейтронов диаметром не более 5 мм, чтобы в месте прохождения луча сквозь тело человека всё умирало примерно за 0,1 сек.
Ищу принцип, на котором можно создать стрелковое оружие будущего. У гауссовок околонулевой КПД, лазеры против пехоты неэффективны от слова совсем, у плазменных очень малая дальность. Нейтронное оружие пока смотрится не плохо, нейтроны легко проходят сквозь любые укрытия, сделать против них эффективную броню сложно (под сложно я имею ввиду что можно, но она будет тяжёлой). Но было бы очень печально, если бы солдат почувствовал что в него попали только через полчаса после попадания.
Аноним 24/06/18 Вск 09:10:45  441484
>>439790 (OP)
Вселенная детерминирована, но детерминированы ли начальные параметры вселенной?
Аноним 24/06/18 Вск 10:19:06  441486
>>441480
>луча нейтронов
Ебать дебил.
Стреляй уж сразу ЧД, хули стесняться.
Аноним 24/06/18 Вск 13:03:45  441493
>>441486
ЧД малых размеров быстро испаряются излучением Хокинга, их не в какой патронник не засунешь.
А википедия говорит что в теории нейтронная пушка возможна, принципы на которых её можно сделать известны, просто мы понятия не имеем, как именно. Примерно как с тем же сингулярном генераторе, который работает на излучении хокинга от мини-ЧД.
Аноним 24/06/18 Вск 14:58:50  441499
>>441493
>принципы на которых её можно сделать известны
>просто мы понятия не имеем
Оно еще и косноязыкое хуйло.
Очередная ЖИСТОКАЯ ГОЛЛАКТЕКА в общем.
Аноним 24/06/18 Вск 16:32:35  441506
>>441499
Если ты видишь то, чего нет (например косноязычие), то ты скорее всего шизик.
Аноним 24/06/18 Вск 18:35:48  441510
Ой, а почему два треда тупых вопросов ? Продублирую.
Допустим необходимо аппроксимировать закон работы некоторого чёрного ящика. Можно подать ему на вход параметры и посмотреть что будет на выходе. Допустим закон найден и получена оценка точности <100 %. Вопрос: как можно проверить не вызвана ли неточность наличием скрытых параметров ? Вот теорема Белла из квантовой физики как раз об этом, а я ищу её мат. основы. Это же на статистике всё основано и наверняка применяется для анализа данных, нейронных сетей и т. п. Вот я ищу самые основы этой статистики без применения в конкретной области.
Аноним 24/06/18 Вск 18:46:36  441512
>>441510
Хм... интересный ты вопрос задал. Надо подумать.
Аноним 24/06/18 Вск 22:47:52  441535
>>441510
У черного ящика кроме скрытых параметров других параметров и нет. Поэтому можно говорить, что оценка точности основывается только на скрытых параметрах и точности измерительных приборов.
Аноним 25/06/18 Пнд 01:24:25  441545
Зачем в век контента потребление нужны учебники?
Запускаешь несколько видео по обучению и правило учить уже не нужно. Я за несколько видео с объяснениями по математики и физики понял больше чем с живым репетитором.
У меня ощущения что никто не хочет делится знаниями, а хочет наёбывать с образованием.
Аноним 25/06/18 Пнд 01:45:54  441548
Почему вы отрицаете детерминизм и бихевиоризм?
Ведь глупо только потому что никто пока не понял квантовую физику.
Если бы не было то была бы анархия и не было бы системы.
Ребёнок убивал бы родителей, не было бы правительства.
Каждый друг друга бы не понимал.
Аноним 25/06/18 Пнд 03:08:59  441557
>>441545
Поверь, как только пойдут сложные темы, ты будешь очень рад именно бумажному учебнику в руках.
Аноним 25/06/18 Пнд 10:32:02  441567
>>441535
Я возможно не к месту употребил чёрный ящик. Т. е. по результатам наблюдений есть таблица в которой в последнем столбце выход функции, а во всех остальных аргументы - какие-то параметры, котрые мы решили просто по логике измерять и на них строить аппроксимацию. А как понять есть ли ещё скрытые параметры, которые стоит тожи измерить и учесть ? Т. е. не ясно расходятся наши предсказания и реальность из-за неточности аппроксимации или потому что мы тупо не учли важный параметр.
Аноним 25/06/18 Пнд 13:52:31  441577
В сосничестве, когда залезал в шкаф, бабка тут же заставляла вылезать, "а то задохнёшься там!". Разве такое возможно?
Аноним 25/06/18 Пнд 14:00:42  441578
>>441577
Зависит от герметичности шкафа
Аноним 25/06/18 Пнд 14:27:19  441579
343218281821475[...].jpg (19Кб, 495x400)
Друзья, школьный вопрос по закону Паскаля.

Можно ли одним пальцем поднять автомобиль, используя сообщающиеся сосуды и соответствующие гидроцилиндры? Просто никогда не сталкивался с этим... Так ли все как на пике?
Аноним 25/06/18 Пнд 14:45:07  441580
>>441578
Ты непонил. Когда воздуха станет не хватать, у человека возникнет желание вылезти, и он вылезет. Иначе бы это был популярный способ роскомнадзора.
Аноним 25/06/18 Пнд 14:46:18  441581
>>441579
А теперь прикинь, на сколько долей миллиметра ты поднимешь автомобиль, сделав движение пальцем 1 метр.
Аноним 25/06/18 Пнд 15:23:58  441582
>>441580
> Ты непонил. Когда воздуха станет не хватать, у человека возникнет желание вылезти, и он вылезет.
А если он запаникует и не сообразит?
Аноним 25/06/18 Пнд 21:14:56  441614
>>441582
До момента возникновения неконтролируемой паники будет много времени, когда станет оче некомфортно находиться в шкафу.
Аноним 25/06/18 Пнд 21:32:53  441617
>>441614
А если от ремня будешь прятаться?
Аноним 26/06/18 Втр 00:14:32  441639
>>439868
Делали же. Шумеры, вавилоняне, египтяне, гиперборейцы, китайцы, славяне самые разные. Что-то сохранилось, что-то нет, но только сейчас ты видишь вместо четких и математически доказаных утверждений что твоя мать - шлюха, а ты должен изобрести машину времени и продвинуть свою цивилизацию из прошлого какую-то несусветную хуйню, которую воспринимаешь как найс картиночки. Проблема не в этом. Любому мало-мальски адекватному человеку, особенно учёному, особенно сознательному тоже хочется продвинуть свою цивилизацию, а может даже свой вид как можно дальше обеспечив его выживание, так что как только машина времени будет изобретена - ее очевидно используют, и прилетят таки, вот только не к тебе. Зачем двигать цивилизацию в прошлом, рискуя проебать собственную сюжетную линию, если можно просто полететь во время, когда человечества ещене было и добывать себе спокойненько ресурсы из прошлого, где они еще есть, допиливая собственную цивилизацию (я опираюсь на концепт что улетая в прошлое ты изменяешь не свою вселенную, а параллельную, так что учить нохчей письменности тебе ни к чему). Как бы то ни было, отсутствие гостей из будущего намекает на то, что человечество не изобретет машину времени до краха цивилизации, или, как минимум, окончательного проеба координат святой терры.
Аноним 28/06/18 Чтв 11:32:54  441835
Сколько можно заниматься в день математикой физикой чтобы не заебало?
Я хочу в течение полугода до марта апреля наверстать всё с 5 класса, чтобы сдать ЕГЭ и поступить в вуз, в школе, колледже про математику и физику не спрашивали просто списывали и давали минимум.
Возможно ли за пол года всю школьную программу выучить?
Ну или за 1.5 года если не смогу за пол года.
21 год кун
Аноним 28/06/18 Чтв 12:04:07  441841
>>441835
Похожий вопрос. В 32 года уже поздно идти в вузик? Сейчас прохожу алгебру 9 класса и чувствую себя дегенератом. Но в моём мирке туповатых дядек могут принять на платные места в средне паршивых технических университетах.
Аноним 28/06/18 Чтв 19:36:26  441874
>>441835
>заниматься математикой в течение полугода
>чтобы сдать ЕГЭ и поступить в вуз
не стоит
только если для себя, разве что
>>441841
>В 32 года уже поздно идти в вузик?
тоже не стоит
если ты в 32 хочешь пойти в вузик, то тебе уже ничего не поможет, не то что рашковузик
Аноним 29/06/18 Птн 13:36:25  441919
Увидел по телеку, что в магазинном мясе(корейка, карбонат, бекон) очень много соли. Можно как-нибудь в домашних условиях перед употреблением ломтика такого мяса избавить его от соли ? Желательно без потери белков и вкусовых качеств(я знаю что без соли вкус будет другой, это ничего).
Аноним 29/06/18 Птн 13:44:38  441920
>>441919
Проще купить кусок мяса. И дешевле, и проще.
Аноним 29/06/18 Птн 13:48:51  441921
>>441920
Ты что ? Тут отрезал и кушаешь, а мясо надо готовить, оно ещё и ужарится. Это очень не факт что проще и дешевле. Как раз в том то и дело, что мясом, котлетой я иногда не наедаюсь, тогда и отрезаю пару ломтиков корейки.
Аноним 29/06/18 Птн 13:50:38  441922
>>441921
>Это очень не факт что проще и дешевле
Немного неочевидный, но факт.
Иначе колбасу было бы невыгодно делать, прикинь? Её же из мяса делают? И она в весе теряет раза в два.

А вообще, лучше не кушай колбасы и обработанное мясо, повышаешь себе риск рака.
Аноним 02/07/18 Пнд 23:07:27  442198
Существуют ли простые вещества-диэлектрики в твердом состоянии в нормальных условиях?
Аноним 02/07/18 Пнд 23:48:14  442199
>>442198
Многие газы например.
Аноним 03/07/18 Втр 01:26:55  442200
>>442198
Стекло и пластмасса подойдет?
Аноним 03/07/18 Втр 17:47:13  442217
Так бле, элементарные частицы, которые имеют предрасположенность складываться в водород, образовались из лучистой энергии, или из-за флуктуаций вакуума? Или из лучистой энергии из-за флуктуаций вакуума?
Или это разные теории?(Из лучистой энергии-тбв, а из флуктуаций-теория флуктуаций, или как она там называется..)
Аноним 04/07/18 Срд 01:21:46  442237
>>442199
>Многие газы например.
>в твердом состоянии в нормальных условиях

>>442200
>Стекло и пластмасса
>простые вещества
Аноним 04/07/18 Срд 02:41:37  442242
>>442237
Бери углерод.
Я не поверю, что человек, который отличает "простое" вещество от "составного". Не может сам себе ответить на вопрос о диэлектриках.
Аноним 04/07/18 Срд 09:43:53  442249
>>442237
Сухой лёд блядь, хули ты такой тугой.
Аноним 04/07/18 Срд 11:51:18  442252
>>442249
Но в нём не нормальные условия, лалка.
Аноним 04/07/18 Срд 14:17:03  442255
>>442252
Это ты ненормальный
Аноним 04/07/18 Срд 15:32:23  442256
>>442242
>Бери углерод.
Графит и алмаз - полупроводники.
Бор - вроде как тоже полупроводник.
Сера и Фосфор - тоже.
А остальное либо газ(и не факт что в твердом состоянии не будет полупроводником), либо проводник.

Потому ж и спрашиваю - возможны ли вообще простые вещества с ярко выраженными свойствами диэлектрика, или для этого вещество должно состоять из разных атомов чтобы появились электрические диполи и прочие
"неровности"? .

>>442249
>Сухой лёд
>CO2
>требует низкой температуры/высокого давления
>хули ты такой тугой.

Аноним 04/07/18 Срд 15:46:55  442257
>>442256
>Графит и алмаз - полупроводники
С алмазом сомневаюсь жутко. Больно уж он ковалентный.
Графит сильно меняет свойства в зависимости от ориентаций слоев.
Аноним 09/07/18 Пнд 12:34:56  442550
1)Всё производство взрывчатых веществ по сути это просто запасание энергии в чём-то, что может её потом быстро отдать?
Все такие производства крайне энергозатратны получается, ровно на количество энергии запасённой в их продукции, плюс кпд запихивание её в эти вещества?

2)Из всего что находится в периодической таблице до железа можно получить энергию синтезом, а из всего что после-распадом?
Всё что после железа-медленно распадается, а то что до-устойчиво и "вечно", относительно того что после?
Аноним 15/07/18 Вск 11:50:31  443089
Что такое iq тест, написано коэффициент интеллекта но что такое интеллект ?
Я думаю что iq знание алгебры математики, ведь когда я учу алгебру математику количество баллов увеличивается на 10-20 баллов.
Почему каждый тест показывает разный результат от 91 за 5 минут до 123 баллов в разное время.
Где настоящий тест пройти
Аноним 16/07/18 Пнд 14:57:58  443255
Почему у меня огромный второй подбородок и как от него избавиться?
рост 187, вес 192
Аноним 21/07/18 Суб 17:01:43  443589
Может ли мозг научиться воспринимать радиоволны?
Гуглится всякая хуита типа вреда излучений, а меня интересует именно восприятие а не вред.
Ну тоесть подаешь на мозг частоту(нет, не с магнетрона), и человек говорит что ее чувствует как-то.

Поидее мозг это сплетение нейронов, в месте контакта может быть что-то типа диода - детектор готов.
Аноним 23/07/18 Пнд 09:44:36  443751
>>443589
Радиоприемник проще и дешевле.


Топ тредов
Избранное